You are on page 1of 79

FRCA primary onexamination 1

Question: 3 of 10
Hypokalaemia is associated with:
True / False
Insulin infusions Correct

Oral Furosemide therapy Correct

Spironolactone therapy Correct

Salbutamol inhaler therapy Correct

Prednisolone therapy Correct

Hypokalaemia is associated with insulin therapy due to the effect of insulin on the cell membrane sodium-potassium
pump causing intracellular shift of potassium. Inhaled salbutamol (unlike IV) is not associated with hypokalaemia.
Prednisolone is capable of producing hypokalaemia due to excessive potassium excretion. Conversely addison's is
associated with hyperkalaemia. Spironolactone is asociated with potassium retention. Thiazide and loop diuretics are
both associated with hypokalaemia.

Question: 4 of 10
Which of the following are correct regarding Desflurane:
True / False
boils at 23C. Correct

is safely used in subjects with malignant hyperpyrexia. Correct

is a fluorinated methyl-isopropyl ether. Correct

prolongs the duration of muscle relaxants. Correct

stimulates the sympathetic system when the inspired concentration is suddenly increased. Correct

Desflurane is a fluorinated methyl, rather than isopropyl ether, inhalation general anaesthetic agent with
low solubility in blood and body tissues, and approximately one-fifth the potency of isoflurane. It has a
boiling point of 22.8°C. The induction of anaesthesia can stimulate the sympathetic system as can change
in inspired concentrations.

Question: 5 of 10
Enoximone:
True / False
acts by blocking Na/K pumps. Correct

increases intracellular cAMP. Correct

causes peripheral vasoconstriciton. Correct

is a type 3 phosphodiesterase anatagonist. Correct

is effective in the treatment of left ventricular failure. Correct


Enoximone is a competitive and selective inhibitor of type III isoenzyme of phopshpodiesterase. Consequenrtly, it
casues increased intracellular cAMP - with vasodilatation and inotropic effects. It is used in cardiac failure. Side
effects include arrhthymias, deranged LFTs and thrombocytopaenia.

Question: 6 of 10
The following reduce blood glucose:
True / False
Diazoxide Correct

Isoprenaline Correct

Tolbutamide Correct

Gliquidone Correct

Glucagon Correct

Tolbutamide and Gliquidone are sulphonylureas, which act by augmenting insulin release and are effective in non-
insulin dependent (type 2) diabetes mellitus. Glucagon is a polypeptide hormone produced by the alpha cells of the
islands of Langerhans, which mobilises hepatic glycogen stores. It can be injected by any route and is used to treat
hypoglycaemia. Diazoxide is an intravenous antihypertensive agent, which is associated with hyperglycaemia and
sodium / water retention. Isoprenaline is a beta-adrenergic agonist (inotropic sympathomimetic) which increases
cardiac output, heart rate and blood glucose levels; it is only available by special order.

Question: 7 of 10
The following drugs inhibit enzymes:
True / False
Acetazolamide Correct

Allopurinol Correct

Piroxicam Correct

Trandolapril Correct

Cimetidine Correct

Acetazolamide is a carbonic anhydrase inhibitor and a weak diuretic. It is used for prophylaxis against mountain
sickness and in the management of glaucoma. Allopurinol inhibits the enzyme xanthine oxidase which is responsible
for urate synthesis and is used in the management of gout. Piroxicam is a non-steroidal anti-inflammatory drug
(NSAID) that inhibits cyclo-oxygenase. Trandolapril is an angiotensin-converting enzyme inhibitor (ACE inhibitor).
Cimetidine is a histamine type-2 receptor antagonist but as a side effect it inhibits the mixed function
oxidases (cytochrome P-450s) and so retards oxidative hepatic drug metabolism (phase 1 reactions).

Question: 8 of 10
Sevoflurane:
True / False
Is a halogenated ether with anaesthetic properties. Correct

Has a minimal alveolar concentration (MAC) of approximately 2%. Correct

At atmospheric pressure it boils at 23ºC. Correct

Is 0.2% metabolised by the liver. Correct


Can cause hepatitis due to compound A formation. Correct

Sevoflurane is a halogenated methyl-isopropyl ether and has a minimal alveolar concentration (MAC)
quoted at between 1.8% and 4%. The most accepted MAC values are approximately 2 - 2.5%. The boiling
point of sevoflurane is 58.8ºC (not 23ºC) and the saturated vapour pressure (SVP) is 23.3 kPa.
Sevoflurane is 2% metabolised by the liver (not 0.2%) but forms no reactive products. It is therefore
considered safe in patients sensitised to trifluoroacetic acid, which is thought to be involved in halothane
hepatitis.

Compound A is formed by the reaction of sevoflurane with dry carbon dioxide absorbers at low gas flows,
and is nephrotoxic not hepatotoxic. The formation of compound A is greater with carbon dioxide absorbers
which contain sodium and potassium hydroxides. The fluoride ion is also produced by the metabolism of
sevoflurane that is nephrotoxic in rats, although the significance to humans is debatable.

Question: 9 of 10
Bronchial tone may be increased by:
True / False
ibuprofen Correct

morphine Correct

ketamine Correct

neostigmine Correct

suxamethonium Correct

Drug induced bronchoconstriction may be a recognised side effect or an idiosyncratic reaction. Inhibition
of prostaglandin synthesis by aspirin and nonsteroidal anti-inflammatory drugs (NSAIDS) like ibuprofen,
may induce an asthmatic attack in susceptible patients. Drugs causing histamine release (morphine,
atracurium and suxamethonium) and those that increase cholinergic activity (neostigmine) may increase
bronchial tone. Ketamine causes bronchodilatation.

Question: 1 of 10
Concerning the central venous pressure (CVP) trace, which of the following
is/are true?
True / False
The C wave represents atrial contraction Correct

The V wave represents ventricular contraction Correct

Third degree heart block causes canon A waves Correct

Tricuspid regurgitation causes large A waves Correct

X descent occurs between the A and C waves Correct

A waves represent atrial contraction,


C waves correspond to the tricuspid valve bulging into the atrium during right ventricle contraction and
V waves represent atrial filling with a closed tricuspid valve.
X descent occurs following the C wave.
Large A waves occur with right ventricular hypertrophy and complete heart block.
Tricuspid regurgitation causes giant V waves.
Question: 2 of 10
Alveolar surfactant:
True / False
helps to prevent alveolar collapse Correct

is decreased in hyaline membrane disease Correct

is increased in hypothyroidism Correct

contains approximately 60% dipalmitoylphosphatidylglycine Correct dipalmitoylphosphatidylcholine

is decreased in the lungs of heavy smokers Correct

Surfactant is a surface tension lowering agent found in the fluid lining the alveoli, which prevents collapse and
pulmonary oedema. It is produced by type II alveolar epithelial cells and it is a mixture of lipids and proteins, in the
following proportions: dipalmitoylphosphatidylcholine 62%; phosphatidylglycine (5%); other phospholipids 10%;
neutral lipids 13%; proteins 8%; and carbohydrate 2%. Surfactant deficiency is the cause of hyaline membrane
disease and there is a decrease in surfactant in the lungs of cigarette smokers. Thyroid hormones increase the
number of inclusions in the type II cells and hyaline membrane disease, due to a deficiency in surfactant, is more
common in infants with hypothyroidism.

Question: 3 of 10
Which of the following statements regarding invasive arterial pressure
monitoring is true:
True / False
arterial cannulae have tapered walls Correct

respond within a frequency range of 5 - 20 Hz Correct 0.5 – 40Hz

the natural frequency is indirectly related to the catheter diameter Correct directly

the optimal damping value is 6.4 Correct 0.64

increased resonance elevates the systolic and lower the diastolic pressures Correct

Arterial cannulae should have parallel walls (not tapered) to minimize the effect on blood flow to the distal
parts of the limb. The arterial pressure waveform can be characterised as a complex sine wave, which is
the summation of a series of sine waves of different frequencies and amplitudes. In order to correctly
display the arterial waveform, the system used to measure arterial blood pressure should be capable of
responding to a frequency range of 0.5 – 40Hz.

The natural frequency is the frequency at which the monitoring system itself resonates and amplifies the
signal and it should be at least ten times the fundamental frequency. The natural frequency is directly
related to the catheter diameter (not indirectly). It is inversely related to the square root of the: system
compliance, length of tubing and the density of the fluid in the system.

Damping can be caused by an air bubble, clot or a very compliant diaphragm and tube. Increased
damping lowers the systolic pressure and elevates the diastolic pressure. The monitoring system should
have an optimal damping value of 0.64.

Resonance can be due to a stiff, non compliant diaphragm and tube. Increased resonance elevates the
systolic and lowers the diastolic pressures.
Question: 4 of 10
Wheezing secondary to left ventricular failure may be associated with which
of the following:
True / False
a fourth heart sound Correct

pulsus alternans Correct

pulsus paradoxus Correct

pleural effusion Correct

a third heart sound Correct

The precise aetiology of wheezing may be difficult to diagnose. Left ventricular failure is characterized by
symptoms of dyspnoea, orthopnoea and paroxysmal nocturnal dyspnoea. Inspiratory crackles
characteristic of pulmonary oedema may be absent in the presence of pulmonary vasoconstriction.

Pulsus alternans is an arterial pulse that is alternately weak and strong, occurring with uniform QRS
complexes. It is an uncommon sign but strongly indicates the presence of left ventricular disease. Third
and fourth heart sounds may be present in left or right ventricular failure. Pleural effusions occur with
severe cardiac failure.

Pulsus paradoxus in a patient with wheeze is suggestive of asthma but it can occur in cardiac diseases
such as constrictive pericarditis and cardiac tamponade. However, these patients may develop
biventricular failure rather than only left ventricular failure. Cardiac tamponade is also a cause of pulseless
electrical activity leading to cardiac arrest.

Question: 5 of 10
The following statements concern the monitoring of neuromuscular function:
True / False
the post-tetanic count (PTC) is inversely related to the neuromuscular block Correct

double burst stimulation (DBS) is used in the detection of phase 2 block Correct

the train-of-four (TOF) stimulation should not be repeated more than once every 60 seconds Correct

fade to 100Hz stimulation occurs even in the absence of neuromuscular blocking agents Correct

a post-tetanic count of one, indicates optimal conditions for trachea intubation Correct

Those who administer neuromuscular blocking drugs require an understanding of neuromuscular function
monitoring, to safely use the drugs and to accurately determine the reversibility of the neuromuscular
block. The train-of-four (TOF) count is the mainstay of clinical monitoring and was designed to detect fade
on repetitive stimulation with non-depolarizing drugs. It involves four stimuli at 2Hz repeated every 10 to
20 seconds. The neuromuscular block is estimated from the number of responses present (TOF count) or
from a comparison of the height of the fourth to the first response (TOF ratio). If TOF stimulation is
repeated more often than once every 20 seconds the magnitude of the responses are equally reduced, so
the TOF ratio is unchanged.

The post-tetanic count (PTC) was developed for use when no response is seen to TOF, single twitch or
tetanic stimulation. It consists of one 50 Hz tetanus for 5 seconds, followed by a 3 second pause and then
single twitch stimulation at 1 Hz. The PTC is the number of responses to single twitch stimulation and is
inversely related to neuromuscular block. Thus, a PTC of 1 indicates optimal intubating conditions
(equivalent to a TOF count of zero).

Double burst stimulation was specifically designed to improve the detection of fade in assessing the
recovery from a non-depolarising block. It involves two short bursts of tetanus at 50 Hz with a 0.75 second
delay. The number of stimuli in each of the bursts has been varied but 3 impulses appears to be most
satisfactory, called DBS 3,3. A phase 2 block occurs with repeated doses of depolarising agent. The block
has features similar to a non-depolarising block (with fade to repetitive stimulation and post-tetanic
facilitation). Even in the absence of neuromuscular blocking drugs, fade can be observed to 100 Hz
stimulation.

Question: 6 of 10
Amiodarone:
True / False
Is a class II antiarrhythmic agent. Correct

Reduces atrioventricular node automaticity. Correct

Has a significant negatively inotropic effect when given orally. Correct

Is a significant cause of abnormal liver function tests on ITU. Correct

Is about 70% protein bound in the plasma. Correct

Class II antiarrhythmics are beta adrenoceptor antagonists. Amiodarone is a class III antiarrhythmic agent,
which reduces atrioventricular node automaticity. An intravenous injection of Amiodarone may have
negatively inotropic effects, but the drug is essentially free of this effect when given by the oral route.

Up to 50% of patients on ITU develop abnormal liver function tests as a consequence of amiodarone
administration. Protein binding is almost 99% in the plasma, which explains the long half life.

Question: 7 of 10
Which of the following features are found in flowmeters on modern
anaesthetic machines in the United Kingdom:
True / False

Oxygen is the first gas to be added to the mixture delivered to the back bar Correct

Have an accuracy of about +/- 4% Correct +/- 2%.

The use of stanic oxide increases the accuracy of flowmeters Correct

The reading of the flowmeter is taken from the midpoint of the bobbin Correct top

Carbon dioxide flowmeters can deliver up to 10litres/minute. Correct up to 500 ml

Flowmeters measure the flow rate of a gas passing through them, and they are individually calibrated for
that specific gas. Calibration is performed at room temperature and at atmospheric pressure. They have
an accuracy of +/- 2%.

The reading of the flowmeter is taken from the top of a bobbin or the midpoint of a ball. A safety feature of
flowmeters is that oxygen is the last gas (not first) to be added (downstream) to the mixture delivered to
the back bar. This prevents delivery of a hypoxic mixture.
Flow measurements become inaccurate when the bobbin sticks to the inside wall of the flowmeter. Stanic
oxide has been used as an antistatic substance, which prevents the build up of static electricity, reducing
the risk of the bobbin sticking to the wall of the flowmeter. The facility for delivering carbon dioxide is found
on some older machines, but the flowmeters are limited to delivering a maximum flow of 500 ml /min. Thus
the delivery of a hypercarbic mixture is avoided.

Question: 8 of 10
An elderly female has a haemoglobin concentration of 9.8 g/dL (11.5-16.5), a
mean corpuscular volume (MCV) of 72 fL (80-96) and a mean corpuscular
haemoglobin (MCH) of 22 pg (27-34). Causes for this picture include:
True / False
Alcoholism. Correct

Iron deficiency. Correct

B12 deficiency. Correct

Acute Haemolytic anaemia. Correct

Acute upper gastrointestinal haemorrhage. Correct

The patient illustrated has an microcytic hypochromic anaemia and causes include: iron deficiency and
thalasaemia. A macrocytic anaemia may be caused by folate deficiency, B12 deficiency and alcoholism,
whereas a normocytic normochromic anaemia may be caused by acute blood loss, haemolytic anaemia,
chronic disorders and leucoerythroblastic anaemias.

Question: 9 of 10
Regarding the syndrome of inappropriate antidiuretic hormone (ADH)
secretion, which of the following statements is/are true?
True / False
Can precede any radiological signs of cancer Correct

Can present with seizures Correct

Is associated with porphyria Correct

Is best treated with spironolactone Correct

When associated with pneumonia implies an underlying diagnosis of bronchial carcinoma Correct

The syndrome of inappropriate antidiuretic hormone (SIADH) secretion is associated with many conditions
including hypothyroidism, bronchial carcinoma, pneumonia, head trauma, subarachnoid haemorrhage,
porphyria and many more.

It is related to an underlying malignancy and may precede any radiological evidence of carcinoma.
However, when it is associated with pneumonia an underlying bronchial carcinoma cannot be assumed.

Seizures are usually a consequence of acute hyponatraemic encephalopathy.

Treatment of SIADH should focus on the underlying cause and the hyponatraemia.

Doxycycline is helpful but spironolactone may make the hyponatraemia worse.


Question: 10 of 10
Regarding carcinoid syndrome, which of the following is/are true?
True / False
Deranged electrolytes are frequently seen Correct

It is associated with left sided cardiac valve lesions Correct

Small bowel carcinoid tumours rarely metastasise to the liver Correct

Tumours arise from enterochromaffin cells Correct

Urinary bradykinin concentrations may be elevated Incorrect answer selected

Carcinoid tumours may secrete


 5 Hydroxytryptamine (5HT)
 Bradykinin
 Histamine
 Substance P or
 Prostaglandins.

The tumours arise in enterochromaffin (argentaffin) cells and produce symptoms when peripheral levels of
5HT, etc, reach high levels.

36% are found in the small bowel and metastasise to the liver before causing the syndrome. They are
also found in the lung, pancreas, large bowel and stomach.

The classic syndrome is of


 Diarrhoea
 Flushing with hypotension
 Telangectasia and
 Bronchospasm.

Less commonly hypertension and right-sided heart valve lesions are seen.

A raised urinary 5-hydroxyindoleacetic acid (5HIAA) on a low serotonin diet is diagnostic.

Electrolyte imbalance and malnutrition is seen when diarrhoea is the primary clinical manifestation.

Question: 6 of 10
Regarding the pharmacological control of phaeochromocytoma, are the
following alpha-adrenergic receptor antagonists?
True / False
Atenolol Correct

Nifedipine Correct

Phenoxybenzamine Correct

Phentolamine Correct

Propranolol Correct
Alpha-adrenergic receptor blockade is particularly important in the treatment of phaeochromocytoma and
should be started before beta blockers.

Alpha blockers include phenoxybenzamine, phentolamine, prazosin and doxazosin. Generally


phenoxybenzamine is preferred as this provides irreversible blockade.

Atenolol and propranolol are both beta-adrenergic receptor antagonists, and nifedipine is a calcium
antagonist.

Question: 7 of 10
Regarding the reporting of adverse drug reactions (ADR) in the United
Kingdom, which of the following statements is/are true?
True / False
A black triangular sign alongside a drug in the British National Formulary (BNF) requires that all ADRs
associated with this drug be reported Correct

ADR reporting is compulsory Correct

Reporting is not required for ADRs associated with vaccinations Correct

There is no requirement to report ADRs with over the counter (OTC) drugs Correct

When submitting a yellow card concerning an ADR, all other drugs taken within the last three months should
also be reported Correct

Reporting adverse drug reactions (ADRs) on the yellow card to the Medicines Control Agency is a
voluntary process, but all side effects noted with newer drugs denoted with a black triangle in the BNF
should be reported.

Even ADRs with OTC therapies and vaccinations should be reported to ensure adequate monitoring and
potentially ascertain whether the manufacturing of certain brands is appropriate. The report should also
contain drugs that the patient has taken (including OTCs) within the last three months to establish
potential interactions.

Question: 8 of 10
The following inhibit Warfarin metabolism:
True / False
Oral contraceptives Correct

Omeprazole Correct

Aspirin Correct

Rifampicin Correct

Metronidazole Correct

The anti-coagulant effects of warfarin may be exacerbated through the inhibition of its metabolism by cytochrome
P450. Omeprazole, Metronidazole, cimetidine and amiodarone may all do this and so the international
normalised ratio (INR) should be carefully monitored. However, Rifampicin induces P450 and so would reduce the
anticoagulant effects of Warfarin (as may other inducers of P450, e.g. anticonvulsants). Oestrogen may reduce the
anticoagulant effect independently of P450, so the metaboism is unaffected.
Question: 9 of 10
In methyl alcohol poisoning:
True / False
Kussmaul's breathing (air hunger) is a prominant feature Correct

Nikethamide may restore consciousness Correct

Papilloedema may be seen Correct

Methyl alcohol is metabolized to acetaldehyde Correct d-formaldehyde and formic acid (not acetaldehyde)

Plasma bicarbonate is very low Correct

Methyl alcohol poisoning causes a metabolic acidosis and Kussmaul's breathing (air hunger) is a prominant feature.
Papilloedema may be seen and the plasma bicarbonate concentration is low. Methyl alcohol is metabolised to d-
formaldehyde and formic acid (not acetaldehyde). Nikethamide does not restore consciousness.

Question: 10 of 10
Low molecular weight heparin:
True / False
exerts its anticoagulant effect by binding with antithrombin Correct

inactivates thrombin more readily than unfractionated heprin Correct

has fewer chains containing the unique pentasaccharide sequence (the binding site) than unfractionated heparin
Correct

has a more potent effect on platelets than does unfractionated heparin Correct

is excreted in the urine Correct

Low molecular weight heparin binds to antithrombin and creates a complex which then binds with and inactivates
factor Xa. Low molecular weight heparin binds and inactivates factor Xa more readily than unfractionated heprin
because the shorter chains are less likely to bind both antithrombin and thrombin. Low molecular weight heparin has
fewer chains containing the unique pentasaccharide sequence (the binding site) than unfractionated heparin (15-
25% versus 33% respectively). Low molecular weight heparin binds less to platelets, endothelium and Von Willibrand
Factor hence it is associated with a reduced bleeding risk. It is excreted in the urine and partly by hepatic metabolism
(though less so than unfractionated heparin) and needs monitoring in renal impairment.

Question: 1 of 10
The treatment of hyperkalaemia includes which of the following?
True / False
Amiloride Correct

Atenolol Correct

Calcium Correct

Magnesium Correct

Sodium bicarbonate Correct

When electrocardiogram abnormalities are present, treatment of hyperkalaemia is an emergency.


The treatment includescalcium chloride
 Dextrose/insulin
 Beta agonists
 Loop diuretics
 Drugs to bind potassium in the gastrointestinal tract
 Dialysis.
The underlying condition must also be sought and treated.
Amiloride, atenolol and magnesium have no role in the management of hyperkalaemia.
Although sodium bicarbonate was historically used for this purpose, it is now not recommended.

Question: 2 of 10
The following can be directly measured by spirometry:
True / False
vital capacity Correct

anatomical dead space Correct

residual volume Correct

total lung capacity Correct

functional residual capacity Correct

A spirometer is a device used for measuring lung volumes either directly or indirectly using dilution
techniques, e.g. helium. It can also be used to calculate flow rates and the basal metabolic rate.
Spirometry provides timed measurements of expired volumes from the lung and is the foundation of
pulmonary function testing. Wet and dry spirometers exist, and with automated equipment it is possible to
interpret more than 15 different measurements from spirometry alone. Forced vital capacity (FVC), forced
expiratory volume in one second (FEV1), FEV1/FVC ratio, and the flow between 25% and 75% of the FVC
are the most clinically helpful indices obtained from spirometry. The anatomical dead space is measured
by Fowler‟s method (single breath nitrogen washout); residual volume and total lung capacity can be
measured using the body plethysmograph or helium dilution; the functional residual capacity can be
measure by nitrogen washout or the helium dilution technique. It should be noted that the helium dilution
technique is performed by the patient breathing air with a known concentration of helium, starting from the
end of normal expiration from a SPIROMETER! The question asks which can be measured directly, thus
the only correct option is vital capacity.

Question: 3 of 10
Laser emissions:
True / False
are polychromatic Correct

are divergent Correct convergent (not divergent)

form coherent waveforms Correct

have a small cross sectional area Correct

the energy is proportional to the photon frequency Correct

Light amplification by the stimulated emission of radiation or LASERs, are being used with increasing
frequency in the treatment of numerous surgical and medical conditions and procedures.
A laser produces and emits an intense beam of entirely monochromatic light or single frequency (not
polychromatic). The beam is convergent (not divergent) with a small cross sectional area and forms
coherent waveforms (in phase). The energy of the radiated laser beam is governed by the following
equation:
E = hV
Where: E is the energy of the photons; h is Planck‟s constant; V is the photon frequency. Hence, energy is
only dependent on the frequency of the photon.

Question: 4 of 10
Total parenteral nutrition (TPN):
True / False
Provides essential and non-essential amino acids Correct

An emulsion of soya bean oil usually provides the fat Correct

Hypophosphataemia is a known complication Correct

Glucose is the only source of carbohydrate Correct

Each gram of nitrogen requires 5 mmol of potassium and 2 mmol of calcium Correct magnesium

The principles of total parenteral nutrition (TPN) involve calculating nitrogen balance, energy, fluid and
electrolyte requirements. The nitrogen component should be given as mixtures of both essential and non-
essential amino acids. 5-6 mmol of potassium and 1-2 mmol of magnesium (not calcium) are required per
gram of nitrogen.

Carbohydrate is usually given as glucose (10 to 50%), but other energy sources e.g. ethanol, xylitol and
sorbitol have been used. Fat is usually administered as 10 or 20% soya bean oil emulsion. Trace
elements and minerals must also be added. Metabolic complications of TPN include: hypophosphataemia,
hypernatraemia, hyperglycaemia and lipaemia.

Question: 5 of 10
The following cause splanchnic vasoconstriction:
True / False
adenosine Correct

angiotensin II Correct

vasopressin Correct

nitric oxide Correct

Noradrenaline Correct

Gut blood flow is affected by the autonomic nervous system, hormonal factors and local mediators.
Modulation of gut blood flow is predominantly mediated through the adrenergic supply and the effect of
both adrenaline and noradrenaline is similar. Vasopressin and angiotensin II are both released in
response to systemic hypotension, but the splanchnic vessels are more sensitive to their vasoconstrictor
effects than systemic vessels. Nitric oxide is involved in maintaining splanchnic vasodilator tone. Inhibition
of nitric oxide synthesis therefore causes vasoconstriction. Adenosine is a vasodilator, which is released in
response to local tissue ischaemia. Therefore, it tends to increase perfusion and reverse ischaemia.
Question: 6 of 10
Regarding high frequency ventilation:
True / False
It is defined as ventilation at more than 10 times the normal rate. Correct four times

There are two main modes. Correct are three modes used (not two)

Uses small tidal volumes of 5-8ml per hour. Correct 1-3ml per kg (not per hour).

Requires the use of armoured tubes due to the rate of ventilation. Correct

The Hayek oscillator should be connected to an uncuffed endotracheal tube. Correct

High frequency ventilation (HFV) uses small tidal volumes delivered at high frequencies, maintaining gas
exchange without barotrauma or other deleterious effects of intermittent positive pressure ventilation
(IPPV). It is defined as ventilation at four times the normal rate (not ten times), and uses tidal volumes of
1-3ml per kg (not per hour). Conventional endotracheal tubes may be used in high frequency ventilation
and there is no necessity to use reinforced or armoured tracheal tubes.

There are three modes used (not two):

 High frequency positive pressure ventilation (HFPPV).


 High frequency jet ventilation (HFJV).
 High frequency oscillation (HFO).

The Hayek oscillator is a non-invasive form of ventilation that uses high frequency oscillation within a
cuirass. The patient that requires ventilatory support does not have to be intubated as they are fitted into a
cuirass, so that it covers them from the axilla to the lower abdomen. It is not connected directly to an
endotracheal tube. The Hayek oscillator works by decreasing and increasing the pressure within the
cuirass, causing inspiration and expiration respectively.

Question: 7 of 10
Regarding the Pin Index System for gas cylinders, the following are correct:
True / False
Air : 1 and 5 Correct

Nitrous oxide: 2 and 5 Correct

Oxygen : 3 and 5 Correct

Cyclopropane: 3 and 6 Correct

Carbon dioxide: 1 and 6 Correct

The pin index system is a non-interchangable safety system used on size E cylinders or smaller. A
specific pin configuration exists for each medical gas on the yoke of the anaesthetic machine, and the
matching configuration of holes is on the cylinder valve block. Therefore, only the correct cylinder can be
attached to a specific yoke. The correct pin index for oxygen is 2 and 5 and for nitrous oxide it is 3 and 5.
Question: 8 of 10
Do the functional consequences of moderately severe mitral stenosis include
the following?
True / False
Hypercapnia Correct

Raised left atrial mean pressure Correct

Reduced glomerular filteration rate Correct

Reduced lung compliance Correct

Reduced pulmonary vascular resistance Correct

Chronic pulmonary venous congestion leads to reduced lung compliance thereby increasing the work of
breathing.
Reduced cardiac output reduces renal perfusion thereby reducing glomerular filtration rate.
Pulmonary vascular resistance is increased due to raised left atrial pressures and inspiratory capacity
hypoxaemia.

Question: 9 of 10
Regarding the lung defence mechanisms, which of the following statements
is/are true/false?
True / False
Only particles less than 10 microns in size can reach the terminal bronchioles Correct 1 micron

Secretory IgA reaches adult levels in the first month of life Correct

Warming and humidifying inspired air occurs proximal to the tracheal bifurcation. Correct

The ciliary beat frequency is usually about 1000 beats per minute Correct

The respiratory tract proximal to the segmental bronchi may contain commensal micororganisms. Correct

The upper airway warms and humidifies the inspired air, and filters particles larger than 1005 microns.
Only particles below about 1 micron can reach the respiratory bronchioles and alveoli.
Intermediate size particles are trapped by the mucociliary blanket, with a normal beat frequency of 1000
per minute. Each ciliated cell has about 275 cilia, and the blanket moves towards the pharynx at 1cm per
minute.
The cough reflex can generate pressures of up to 300 mmHg, with flow rates of up to 5-6 litre /second.
Secretory IgA makes up the principal antibody in respiratory secretions and reaches adult levels in the first
month of life. Two molecules of IgA combine with a secretory component to yield a dimer which is highly
resistant to digestion by proteolytic enzymes. When lung inflammation occurs, IgG and IgM can also be
found.
The phagocytic ability of alveolar macrophages and the mucociliary mechanism are impaired by
1. Alcohol
2. Cigarettes
3. Hypoxia
4. Starvation
5. Chilling
6. Steroids
7. Nitrogen dioxide
8. Ozone
9. Hyperoxia
10. Diamorphine.
Antibacterial killing of macrophages is decreased by
 Acidosis
 Uraemia
 Acute viral infections (especially rubeola and influenza)
and gases such as
 Sulphur
 Nitrogen dioxide
 Ozone
 Chlorine
 Ammonia
 Cigarettes.
Mucociliary clearance is reduced by
 Hypothermia
 Hyperthermia
 Opiates
 Hypothyroidism.

Question: 10 of 10
A drug with a high first pass metabolism:
True / False
is excreted largely unchanged in the urine Correct

can only be administered parenterally Correct

is extensively metabolised by the liver Correct

has a high extraction ratio Correct

the extraction ratio is reduced in chronic liver disease Correct

 First pass metabolism of drugs that are usually given orally predominantly occurs in the liver. The
gastrointestinal absorption of these drugs is usually excellent (they may be lipo or hydrophilic), and
can be administered orally, providing the dose is increased, e.g. oestrogens, morphine sulphate
and isosorbide dinitrate.

 The extraction ratio is dependent upon liver function and is usually high, i.e. more of the drug is
removed following first pass. Thus, the extraction ratio is reduced in chronic liver disease.

 First pass metabolism can also occur in the gut. Examples include insulin and benzylpenicillin but
these are not usually given orally. Other drugs are partly metabolised in the gut and the liver.

Question: 1 of 100
Choose whether the following are true or false:
True / False
A. Nil by mouth also normally includes oral medications. Correct

B. Patients should generally be encouraged to mobilise early post-operatively. Correct


C. Using low molecular weight heparin post-operatively excludes pulmonary emboli as a cause of chest pain.
Correct

D. Following abdominal surgery, regular deep breathing and a sufficient cough reduces the incidence of post-
operative chest infections. Correct

F. Adequate post-operative analgesia reduces physiological stress and improves outcomes. Correct

 A. Except for major gastrointestinal surgery. oral medications can be continued.

 B. This will reduce the risk of deep vein thrombosis (DVT) and pulmonary embolism (PE).

 C. It will reduce the incidence of PE and DVT but not remove all risk.

 D. Post-operative atelectasis is common and can be improved by physiotherapy and breathing


exercises.

 E. Less endogenous catacholamines leads to less cardiac workload and hence better outcomes.

Question: 2 of 100
Which of the following is/are true regarding the oxidation of pyruvate to
carbon dioxide?
True / False
Can occur under anaerobic conditions Correct

Involves intermediates that are also involved in amino acid catabolism Correct

Is impaired in thiamine deficiency states Correct

Is regulated by the concentration of acetyl CoA in the cell Correct

Occurs exclusively in mitochondria Correct

Altough pyruvate can be broken down by aerobic and anaerobic respiration, oxidation requires oxygen
and hence cannot occur under anaerobic conditions.

This takes place in the mitochondrion in eukaryotic cells, but at the cell membrane in prokaryotic cells.

Acetyl CoA, a product of the pyruvate dehydrogenase reaction, is a central compound in metabolism.

Thiamine is involved as a cofactor in numerous enzymes, and is essential in every cell for adenosine
triphosphate (ATP) production via the Krebs cycle.

Question: 3 of 100
With regard to the DNA structures, are the following statements true?
True / False
Adenine (A) and guanine (G) are purine bases Correct

Each amino acid may be coded by more than one codon Correct

Each DNA strand has a pentose-phosphate backbone with projecting bases Correct

Guanine (G) always pairs with thymidine (T) and adenine (A) with cytosine (C) Correct
There are 64 possible codons Correct

Pyrimidine bases are thymidine (T) and cytosine (C).

G always pairs with C and T with A.

Question: 4 of 100
With reference to capillaries, which of the following is/are true?
True / False
Capillaries in the dermis actively constrict in response to sympathetic stimulation. Correct

Cerebral capillaries have microscopically visible 'leaks'. Correct

Fenestrations are present in the renal glomerular capillaries. Correct

The diameter of a capillary in skeletal muscle is usually about twice the diameter of a red blood cell. Correct

When skeletal muscle is active, more capillaries are open than when it is at rest. Correct

Capillaries link the arterioles and venules and are involved in the delivery of blood to the cells themselves.
Therefore capillaries permit the leakage of plasma through fenestrations.

The ability of blood to flow through these capillaries is closely controlled by arteriolar tone. They do not
have smooth muscle themselves. Hence exercise can stimulate greater opening up of the capillary beds.

The diameter of a capillary is 5 - 10 microns. Erythrocyte diameter is 6 - 8 microns.

Question: 5 of 100
Laryngoscopes:
True / False
A left-sided Macintosh blade in routinely used in adults Correct

The McCoy laryngoscope is based on the standard Robertshaw's blade Correct

The polio blade is at an angle of 140 degrees to the handle Correct

The tip of a Miller blade is designed to be placed into the vallecula Correct

The Wisconsin and Seward are examples of straight blade laryngoscopes Correct

Laryngoscopes are used to perform direct laryngoscopy and can be classed as curved or straight,
according to the shape of the blade.

Straight blade laryngoscopes include the following types - Miller, Soper, Wisconsin and Seward.

They are designed to be advanced over the epiglottis which is then lifted in order to view the larynx.
Straight blades are commonly used for intubating neonates and infants but can be just as easily be used
in adults.

Curved bladed laryngoscopes (not straight) are designed so that the tip is placed into the vallecula -
examples include the Macintosh, polio and McCoy.
The standard Macintosh blade used in adults is the right-sided version. The left-sided blade may be used
in patients with facial deformities that make the use of the standard blade difficult.

The Macintosh polio blade is at an angle of 120 degrees to the handle, and was designed to intubate
patients in the iron lung.

The McCoy laryngoscope is based on the standard Macintosh blade (not Robertshaw's), with a lever
operated hinged tip, which can improve the view at laryngoscopy.

Question: 6 of 100
Do the following drugs have a role in the premedication of patients prior to
surgery?
True / False
Atropine Correct

Chloral hydrate Correct

Glycopyrrolate Correct

Ketamine Correct

Metoclopramide Correct

Ketamine given intramuscularly is effective at sedating patients with mental retardation, dementia or
uncooperative patients; however the issue of consent for surgery must be established prior to
administering the drug. An intravenous induction is usually possible when the level of consciousness has
reduced.

Chloral hydrate given orally is a useful sedating premedication used in paediatric practice (smaller children
and infants) and can provide acceptable conditions for either an intravenous or inhalational induction.

Atropine and glycopyrrolate are both anticholinergic drugs and have antisialogogue properties. Atropine is
administered to infants and neonates prior to anaesthesia and surgery (particularly ENT and ophthalmic
surgery) primarily to reduce the risk of inducing a bradycardia.

Glycopyrrolate has a greater drying effect on airway secretions than atropine, and it is given intravenously
prior to topically anaesthetising the airway for an awake fibreoptic intubation.

Metoclopramide increases the rate of gastric emptying and it is often given to patients at risk of
regurgitation and aspiration perioperatively.

Question: 7 of 100
Are pre-operative patients premedicated for the following reasons?
True / False
To cause sedation Correct

To decrease gastric pH Correct

To increase vagal tone Correct

To reduce anxiety Correct

To reduce salivation and airway secretions Correct


The objectives of premedicating patients pre-operatively are to
 Reduce anxiety and fear
 Reduce secretions and salivation
 Produce sedation and amnesia
 Reduce postoperative nausea and vomiting
 Reduce the volume and increase the pH (not decrease the pH) of gastric contents
 Attenuate vagal reflexes (not increase vagal tone) and sympathoadrenal responses
 To enhance the hypnotic effect of general anaesthesia.

Question: 8 of 100
Do methods of sterilisation include the following?
True / False
Cellulose filtration Correct

Dry heat at 134°C for three minutes Correct

Gamma irradiation Correct

Moist heat at 121°C for 15 minutes Correct

Steam at 80°C and subatmospheric pressure Correct

Moist heat (for example, autoclave) is a method of sterilisation and the process kills all organisms and
spores. It should be applied at
 122°C at 1 atm for 30 minutes
 126°C at 1.5 atm for 10 minutes, or
 134°C at 2 atm for three minutes

to sterilise non-heat sensitive surgical instruments.

Steam is used to increase the temperature so dry heat needs to be at a higher temperature for a longer
time, for example, 160°C for one hour, to have a similar effect to moist heat.
Dry heat is used to sterilise glassware and powders.

Steam at low pressure and 80°C will only disinfect instruments, and it does not destroy spores. The
addition of formaldehyde will cause sterilisation, and is used to sterilise heat sensitive equipment such as
cannulae.

Gamma irradiation is emitted from a cobalt-60 source or an electron accelerator, and is an expensive but
effective means of sterilising plastics prostheses.

Cellulose filtration is used to sterilise heat sensitive fluids but it may not remove viral particles

Question: 9 of 100
Which of the following is/are true regarding 'HELLP' syndrome?
True / False
It is related to pre-eclampsia Correct

It may occur in non-pregnant patients Correct

Lower abdominal pain is a characteristic feature Correct


Patients always have low platelets Correct

The 'H' stands for hypertension Correct

"HELLP"syndrome is a severe life-threatening condition that occurs in pregnancy.

It is a mnemonic which stands for: Haemolysis,Elevated Liver enzymes andLow Platelet count.

Variations exist and the diagnosis does not require all the manifestations.

It is related to pre-eclampsia but the patient may be normotensive.

Right upper quadrant pain is a characteristic finding (not lower abdominal pain).

This acronym stands for:

H - haemolysis EL - elevated liver enzymes LP - low platelets

This syndrome can occur in severe pre-eclampsia. It may be a cause of fitting in a pre-eclamptic patient
after the placenta has been delivered.

This syndrome may be life-threatenening. Hepatocellular dysfunction leads to enzyme release and
necrosis and swelling. This can cause epigastric pain, and, in extreme circumstances, rupture and
exsanguination.

 HELLP (Hemolysis, elevated liver enzymes and low platelet count) syndrome
o cause of thrombocytopenia in pregnancy in about 20% of cases
o maternal platelet count returns to normal within 3-5 days of delivery
o HELLP syndrome is a variant of preeclampsia (1)
o platelet count is usually below 100 × 10^9/l)
o up to 3% of HELLP cases may result in maternal mortality
o stillbirth occurs in up to 20% of cases - particularly as a result of placental abruption and
preterm delivery

Question: 10 of 100
Intra-arterial blood pressure monitoring:
fluid-filled tubing conducts the intravascular pressure wave from the catheter tip to the transducer Correct

non-pressurised fluid is infused through the catheter Correct

the transducer should be at the same height as the catheter insertion point Correct

air bubbles cause a hyper-resonant trace Correct

shortening the lengths of tubing has a dampening effect Correct

 Invasive arterial pressure monitoring provides beat-to-beat information with sustained accuracy.
The intravascular pressure wave is conducted from the catheter tip, situated in the arterial lumen,
to the transducer along fluid-filled tubing. Pressurized fluid is infused through the catheter
continuously. The transducer is usually a strain gauge variable resistor which is connected to an
amplifier and oscilloscope.

 The transducer should be placed at the height of the left atrium at all times. Air bubbles and long
catheter tubing, have the effect of dampening the trace (waveform appears rounded). Increased
damping usually lowers the systolic pressure elevates the diastolic pressure.
Question: 11 of 100
Do clinical manifestations of a pulmonary embolus (PE) in childhood include
the following?
Bradycardia Correct

Chest pain Correct

Fever Correct

Haemoptysis Correct

Reduced central venous pressure Correct

The clinical manifestations of a pulmonary embolus (PE) include:


 Dyspnoea
 Tachypnoea
 Tachycardia (not bradycardia)
 Chest pain
 Cough
 Haemoptysis and
 Fever.

The central venous pressure is usually elevated.

Usually occurs in adolescents,

Back pressure from raised right-sided pressures causes elevated JVP.

Question: 12 of 100
Regarding upper gastrointestinal haemorrhage, which of the following
statements is/are true?
True / False
Dexamethasone should be injected into a bleeding gastric ulcer Correct

Endoscopy should be performed Correct

Endotracheal intubation may be required Correct

It may occur from below the ligament of Treitz Correct

May be caused by enalapril Correct

Severe upper gastrointestinal haemorrhage is diagnosed by haematemesis or the presence of blood in the
gastric aspirate. It can be life-threatening and requires fluid resuscitation, including intubation if
consciousness is reduced.

The ligament of Treitz separates the upper from lower gastro-intestinal tract; thus by definition an upper
gastrointestinal bleed must occur above the ligament (not below).

Treatment includes endoscopy and injection of the bleeding points with epinephrine (not dexamethasone).
Surgey may be required.
Enalapril is associated with peptic ulceration according to the BNF and so gastrointestinal haemorrhage is
a risk.

Question: 13 of 100
In paediatric resuscitation:
True / False
the first shock from the defibrillator should be set at 5 Joules /kg Correct

a cardiac arrest in a child is usually precipitated by a respiratory problem Correct

the correct dose of atropine is 0.1mg/kg Correct

during an asystolic arrest in a 8kg infant, the first dose of adrenaline is 0.8mg Correct

1mmol/kg of sodium bicarbonate is routinely administered during a paediatric cardiac arrest Correct

Primary cardiac arrest is rare in children with structurally normal hearts, and disorders of the respiratory
tract are frequently responsible for precipitating paediatric cardiac arrests. Familiarity with the principles,
equipment and doses of the drugs used in resuscitation is essential. A setting of 4 Joules /kg should be
used for the shocks during defibrillation (not 5 Joules/kg).

Adrenaline or epinephrine is used in the management of both shockable rhythms (e.g. ventricular
fibrillation and pulseless ventricular tachycardia) and non-shockable rhythms (e.g. asystole and pulseless
electrical activity). The dose of adrenaline used in a cardiac arrest is 10 micrograms per kg (0.1ml/kg of
the 1 in 10,000 solution) given intravenously or intraosseously. Thus for an 8kg infant the correct initial
dose of adrenaline is 80 micrograms (not 0.8mg).

Atropine has a limited role in paediatric resuscitation and the correct dose is 20 micrograms per kg (not
0.01 mg/kg).

8.4% sodium bicarbonate does have a role in paediatric resuscitation and the dose is 1 mmol/kg.
However, it is not used routinely during a cardiac arrest and its administration is usually guided by the
results of the arterial blood gases.

Question: 14 of 100
In a venous air embolism in an adult, which of the following statements is/are
true?
True / False
Capnography provides a late means of detection Correct

Cardiovascular collapse can occur with a volume of 10 ml Correct

Electrocardiograph (ECG) changes are common Correct

Pulmonary arterial pressure (PAP) is increased Correct

The 'mill-wheel murmur' is an early sign Correct

Air can enter the circulation through open veins and sinuses at the site of surgery (above the level of the
heart) and via venous catheters. The classic 'mill-wheel murmur' is a late insensitive sign (not early) and
usually requires a volume of 1.5 to 4 ml/kg.
However, cardiovascular collapse may precede this sign, and has been reported with less than 1 ml/kg of
air (not 10 ml).

Capnography provides an early indication (not late) of air embolism by a fall in the end-tidal CO2. This is
due to an increase in dead space caused by the bubbles in the pulmonary bed.

Pulmonary artery pressure (PAP) increases following air embolism because of the obstructed pulmonary
vascular bed. It returns to pre-embolic levels as the embolus is absorbed and cleared.

The ECG changes usually only occur with significant air embolisms and can range from signs of right
ventricular strain and tachycardia to life- threatening arrhythmias.

Question: 15 of 100
In hypovolaemic shock:
the stroke volume is increased Correct

the systemic vascular resistance is decreased Correct

the heart rate is decreased Correct

the central venous pressure is decreased Correct

the pulmonary capillary wedge pressure is decreased Correct

Hypovolaemic shock occurs when the intravascular volume is depleted as a consequence of blood or fluid
loss. The commonly encountered haemodynamic findings include: a decrease in stroke volume, cardiac
output and filling pressures (i.e. central venous pressure and pulmonary capillary wedge pressure); and an
increase in heart rate and systemic vascular resistance.

Question: 16 of 100
The following are recognized complications of invasive arterial pressure
measurement in an adult:
True / False
anticoagulation Correct

fluid overload Correct

compartment syndrome Correct

pneumothorax Correct

pseudoaneurysm Correct

Complications of arterial cannulation include: haematoma formation, distal ischaemia, infection,


embolization, pseudoaneurysm, arteriovenous fistula, compartment syndrome, and blood loss.

Flush volumes must be limited in children otherwise fluid overload may occur and replacing the pressure
bag with a syringe pump may provide some protection against such an occurrence.
Question: 17 of 100
In acute renal failure, are the following indications for renal replacement
therapy?
True / False
A plasma sodium of 162mmol/l Correct

Hyperkalaemia Correct

Metabolic alkalosis Correct

Pulmonary oedema unresponsive to furosemide Correct

Uraemic pericarditis Correct

Criteria for initiation of renal replacement therapy include


 Anuria
 Urea >35mmol/l
 Creatinine >400µmol/l
 K+ >6.5mmol/l
 Fluid overload
 Metabolic acidosis
 Na+ <110 or >160mmol/l and
 Uraemic complications.

Question: 18 of 100
Antibiotics:
True / False
Piperacillin and ciprofloxacin are suitable antibiotics in treating infections due to Pseudomonas aeruginosa.
Correct

Lipid formulations of amphoteracin B are less nephrotoxic than conventional amphoteracin B. Correct

Amphoteracin B is not significantly protein bound. Correct

Nucleoside analogues are important in the treatment of HIV infection. Correct

The quinolones are DNA gyrase inhibitors. Correct

Amphoteracin B is highly protein bound and penetrates body tissues poorly. The toxicity is reduced in lipid
formulations of amphoteracin B. Ciprofloxacin and norfloxacin are both quinolones which prevent bacterial
DNA supercoiling by their action on DNA gyrase, and are suitable antibiotics in treating infections due
toPseudomonas aeruginosa.

Nucleoside analogues inhibit reverse transcriptase and are one of the main classes of drugs used in
human immunodifficiency virus (HIV) infection. Other drugs include protease inhibitors and non-nucleoside
reverse transcriptase inhibitors, which are given in combination in order to reduce side effects.
Question: 19 of 100
The following statements are true regarding infective agents:
True / False
All contain nucleic acids. Correct

Viruses are able to reproduce without infecting a multicellular organism. Correct

The human immunodifficiency virus (HIV) is a DNA virus Correct

CD4 can be used as an indication of HIV disease progression Correct

Co-trimoxazole is the treatment of choice for pneumocystis carinii pneumonia (PCP) in HIV positive patients.
Correct

Prions are proteins and contain no nucleic acids. Bacteriophages are viruses which infect bacteria. The
human immunodifficiency viruses belong to the lentivirus class of RNA viruses (not DNA). Reverse
transcriptase is the enzyme which enables the use of RNA as a template for DNA (this is the opposite to
the normal chain of events) and was derived from these viruses.

Co-trimoxazole is a mixture of Sulphamethoxazole and Trimethoprim and is particularly useful in


pneumocystis pneumonia.

Question: 20 of 100
Concerning urinalysis on patients with acute renal failure, are the following
associations true?
True / False
A high urine:plasma osmolality ratio in intrinsic renal failure Correct

High urinary chloride concentration in intrinsic renal failure Correct

High urinary osmolality in pre-renal failure Correct

High urinary urea and creatinine concentrations in pre-renal failure Correct

Red cell casts are absent in pre-renal failure Correct

Pre-renal causes of acute renal failure (ARF) are associated with the following urinary findings:
 Low urinary sodium and chloride concentration (<20 mEq/l)
 High urinary urea and creatinine concentration (>20 mEq/l)
 High urine osmolality (>400 mosmol/kg)
 High urine:plasma osmolality ratio (>1.8)

Intrinsic causes of acute renal failure (ARF) for example, acute tubular necrosis are associated with the
following urinary findings:
 High urinary sodium and chloride concentration (>40 mEq/l)
 Low urinary urea and creatinine concentrations
 Low urine osmolality (<350)
 Low urine:plasma osmolality ratio (1.2)

It is important to note that the findings listed above may change when pre-renal causes of ARF become
superimposed on chronic intrinsic renal failure or following diuretic therapy.

The presence of red cell casts suggests acute glomerular damage.


Epithelial casts occur in acute tubular damage and forms of acute glomerulonephritis.

White cell casts appear in pyelonephritis.

Granular casts may indicate tubular damage, but they can also be found in the urine of normal individuals.

Hyaline casts may be found during any febrile illness and after loop diuretic therapy.

Question: 21 of 100
Hypertension is associated with the following conditions and features:
True / False
Acute intermittent porphyria and motor neuropathy Correct

Congenital adrenal hyperplasia Correct

Adult polycystic kidney disease and an autosomal dominant mode of inheritance Correct

Cushing's syndrome, with sodium retention and potassium depletion Correct

Conn's syndrome, with hyperkalaemia and metabolic alkalosis Correct hypokalaemia not hyperkalaemia

Primary (or essential) hypertension accounts for over 90% of cases hypertension.
Secondary causes can be broadly classified as follows:
 Renal: chronic glomerulonephritis / pyelonephritis, polycystic kidney disease, tumour, renal
artery stenosis, obstruction, analgesic nephropathy;
 Endocrine: Cushing‟s syndrome, Conn‟s syndrome, congenital adrenal hyperplasia,
phaeochromocytoma, acromegaly;
 Drugs: oral contraceptive pill, corticosteroids, monoamine oxidase inhibitors;
 Pregnancy: pre-eclampsia / eclampsia;
 Others: aortic coarctation, acute intermittent porphyria.

In Cushing‟s syndrome the elevated glucocorticoid levels, which have weak intrinsic mineralocorticoid
properties, results in potassium depletion and sodium retention. Conn‟s syndrome (primary
hyperaldosteronism) causes hypernatraemia, hypokalaemia and a metabolic alkalosis.

Question: 22 of 100
Which of the following is/are true regarding a reduced arterial pCO 2?
True / False
Increases cerebral blood flow Correct

Leads to a more alkaline urine Correct

Occurs at altitudes over 2,500 metres Correct

Occurs in normal pregnancy Correct

Reduces blood pH Correct

Reduced pCO2 may be seen in


 The later stages of pregnancy
 At altitude
 In type 1 (emphysema, pulmonary embolism [PE]) and type 2 respiratory failure (muscle
disease, chronic bronchitis)
 In methaemoglobinaemia.

Increased pCO2 increases cerebral blood flow and also increases acidity (respiratory acidosis) which in
turn leads to a more acid urine.

Question: 23 of 100
A 95 % confidence interval:
True / False
can only be used in parametric data Correct

it is a test of the null hypothesis Correct

it is calculated at ± 1.96 times the standard error of the mean Correct

is useful when comparing data with another population Correct

if zero difference lies within the 95% when comparing two groups to a treatment, it indicates the treatment has
no effect Correct

95% confidence intervals can be used for both distributional and distribution-free data. A 95% confidence
interval looks at the range of values within which we are 95% confident that the true population parameter
lies. Therefore, using the above formula, if we were to repeat the experiment many times, the interval
would contain the true population mean on 95% of occasions. Confidence intervals increase the accuracy
when comparing means with another population by looking at the spread of differences. A wide
confidence interval indicates that the estimate is imprecise and if the 95% CI crosses zero, which is the
value of no difference, then it indicates that the treatment has no effect.

Question: 24 of 100
Is it true that the prevalence rate of a disease has the following features?
True / False
It can be estimated from a cross-sectional study Correct

It can be used to determine the health needs of a community Correct

It is dependent on the duration of illness Correct

It is dependent on the incidence of the disease Correct

It measures all the current cases in the community Correct

Prevalence depends on the number of individuals who contract the disease in a particular time period.
Because it looks at the number of individuals with a disease at a given point in time, or within a defined
interval, if a patient has recovered from the illness in that duration, then they would not be included in the
prevalence rate.

It is expressed as a proportion. As cross-sectional studies are effectively a snap-shot they can be used to
estimate the proportion of people with a disease at that time and thus the point prevalence.

Prevalence is one measure that can assess the health needs of a community.
Question: 25 of 100
In atrioventricular (AV) conduction disorders, which of the following is/are
true/false?
True / False
Broad QRS complexes are required for a diagnosis of third degree (complete) heart block Correct

First degree heart block is associated with sudden death Correct

In type 2 second degree heart block the P-R interval is constant Correct in the conducted impulses only

Third degree (complete) heart block is associated with dystrophia myotonica Correct

Type 1 second degree heart block (Wenkebach) is usually asymptomatic Correct

First degree heart block is usually due to a delay in conduction at the atrioventricular (A-V) node, which
causes prolongation of the P-R interval. The delay may also be due to abnormal atrial or bundle of His
conduction. It is usually asymptomatic but can progress to second and third degree heart block.

Second degree heart block is due to intermittent failure of A-V conduction. In type 1 (Wenkebach) the P-R
interval increases progressively until there is failure of A-V conduction. The sequence is then repeated. It
is usually due to A-V node pathology.

Although it rarely causes symptoms, it can progress to type 2 second degree heart block or complete
heart block, so pacemaker insertion should be discussed.

Type 2 second degree heart block results in a variable ratio of non-conducted to conducted impulses
(commonly 2:1). The P-R interval of the conducted impulses is constant. It is usually due to infranodal
pathology and is associated with Stokes-Adams attacks and sudden death.

Third degree heart block (complete A-V block) may be congenital or acquired. Acquired causes include
dystrophia myotonica, rheumatic fever, connective tissue disorders and drug toxicity. It can result in
syncope, cardiac failure and sudden death.

If the heart block is situated in the A-V node and impulses are initiated from the bundle of His, the
resultant QRS complexes are of normal duration.

Broad complexes are produced if the impulses are generated from the right or left bundle branches, but
are not required for diagnosis of third degree heart block.

Question: 26 of 100
The following are true about evidence-based medicine:
True / False
Combines clinical expertise and external evidence Correct

Does not involve health economic assessment Correct

Is restricted to randomised placebo-controlled trials Correct

Is used to cut down waiting list Correct

Tries to rely on objective measurements of disease outcomes Correct


Health economic assessment is central in evidence-based medicine, especially when making judicious
use of current best evidence to make decisions. It includes all trials but one must critically appraise the
methods in order to assess the validity of the evidence.

It is used to make decisions about the care of individual patients. Objective measurements of disease
outcome eliminate bias and are more scientific relative to subjective measures and are therefore
applicable to the practice of evidence-based medicine.

This combination results in the conscientious, explicit and judicious use of current best evidence in making
decisions about the care of individual patients.

Question: 28 of 100

Causes of hyponatraemia include:


True / False
Vomiting Correct

Diarrhoea Correct

Hypertonic saline Correct

Diabetes insipidus Correct

Cirrhosis Correct

Assessment of the volaemic status of the patient, allows the causes of hyponatraemia to be sub-divided
into three categories:
 Euvolaemia e.g. syndrome of inappropriate anti-diuretic hormone (SIADH), high water intake,
hypothyroidism.
 Hypovolaemia e.g. diarrhoea, vomiting, diuretics, renal tubular dysfunction.
 Hypervolaemia e.g. cirrhosis, heart failure.

Diabetes insipidus and the administration of hypertonic saline cause hypernatraemia.

Question: 29 of 100
May intraoperative blood transfusions be reduced by the following?
True / False
A high proportion of hypochromatic erythrocytes Correct

A high starting packed cell volume Correct

A low central venous pressure Correct

A low mean arterial pressure Correct

An epidural block Correct

Intraoperative blood transfusions may be reduced by maintaining a low mean arterial pressure and a low
central venous pressure and through the use of an epidural.

A high packed cell volume has been shown to reduce transfusion requirements in orthopaedic surgery.

A high percentage of hypochromatic erythrocytes indicates functional iron deficiency with an increased
likelihood of transfusion requirement.
Question: 30 of 100
Is it true or false that the functional residual capacity is the following?
True / False
Approximately 2.5 litres Correct

Decreases on lying down Correct

Is measured by using helium as the tracer gas Correct

Is 60% of vital capacity Correct less than 50%

The tidal volume plus the expiratory reserve volume Correct

The functional residual capacity (FRC) is the residual volume plus the expiratory reserve volume, is
approx 2.5l and represents less than 50% of vital capacity.

It is decreased when supine, increases on standing and varies in relation to height and body habitus.

Expiratory reserve can be measured directly and residual volume calculated by helium.

Question: 31 of 100
Ketamine:
True / False
Is an alkaline solution Correct

May be indicated in the management of status epilepticus Correct

It is contra-indicated for epidural administration Correct

Decreases myocardial oxygen demand Correct

Has an R (-) enantiomer which is less psychoactive Correct

Ketamine is an intravenous anaesthetic agent, derived from phencyclidine (angel dust). It is not a true
intravenous induction agent, as anaesthesia is not induced within one arm-brain circulation time, since
onset takes more than 30 seconds. It produces a state of “dissociative anaesthesia” – analgesia with light
sleep, due to increased thalamic and limbic activity, with dissociation from cortical centres. An acid (pH
3.5-5.5) in solution, it is poorly (12%) bound to albumin, with an elimination half life of 3 hours. It may be
administered PO, IM, IV, PR, nasally as well as by both the intrathecal and extradural routes (10mg/ml
preparation only). The 50mg/ml and 100mg/ml preparations contain 1/10,000 benzethonium chloride as a
preservative. Ketamine increases sympathetic outflow, leading to a tachycardia and increased cardiac
output which together will increase myocardial oxygen demand. For this reason caution is advised when
administered to patients with ischaemic heart disease. Its profound bronchodilator action makes ketamine
a useful adjunct in the management of intractable status asthmaticus (not status epilepticus). The S (+)
enantiomer is 2-4 times more potent than the R (-) and less psychoactive, reducing the incidence of
hallucinations, it also has a shorter duration of action.
Question: 32 of 100
Which of the following stimulate contraction of the pregnant uterus?
True / False
Ergotamine Correct

Progesterone Correct

Ritodrine Correct

Suxamethonium Correct

Vasopressin Correct

Uterotonics include oxytocin, prostaglandins and ergot alkaloids.

Ritodrine is a tocolytic agent used to treat premature labour.

Progesterone also has tocolytic effects.

Vasopressin in pharmacological doses does not appear to have any uterotonic effects and is used by
women with diabetes insipidus during pregnancy.

Question: 33 of 100
Which of the following are tocolytic?
True / False
GTN Correct GTN = nitroglycerin

Salbutamol Correct

Progesterone Correct

Propofol Correct

Suxamethonium Correct

Tocolytic drugs, inhibitors of uterine contraction, include GTN, alcohol, magnesium sulphate, ritodrine,
salbutamol, nifedipine and NSAIDs.

Progesterone in high concentrations also has some tocolytic activity and promotes the relaxant effects of
more conventional tocolytics.

Question: 34 of 100
Life table analysis:
True / False
Can only be used to express mortality risk in a population Correct

Use prevalence data from the population Correct

Can be used to calculate relative risk Correct

Can be adjusted to allow for co-variables Correct

From two groups can be compared by calculation of a chi-squared statistic Correct


Life table analysis is used in various contexts to follow a population until certain end points occur. Death is
a suitable end point but development of disease or disability are also suitable.

For example, the mortality in groups of smokers and non-smokers could be collected over a period of time
and survival plotted as a function of time. The development of retinopathy in diabetics, or the time from
treatment of multiple sclerosis patients treated with a interferon or placebo to next relapse would also be
suitable for life table analysis.

These incidence data are best collected prospectively.

Life tables from two groups can be compared by calculating a chi-squared statistic (Mantel-Haenszel
procedure or log rank method). Relative risk can also be calculated from such data.

Mathematical models can be applied to life table data to adjust for confounding variables (co-variables).
An example of this is the Cox proportional hazards model.

Question: 35 of 100
The dinamap automated blood pressure monitoring devive:
True / False
utilises a double cuff oscillotonometric system. Correct single cuff oscillotonometric

Should be positioned at the same level as the patient. Correct

is accurate even in rhythm disturbances. Correct

provides a more accurate measure of the diastolic than systolic pressure. Correct

is more accurate than a manual oscillotonometer. Correct

Dinanap offers continuous monitoring of BP using single cuff oscillotonometric measurement of systolic, diastolic and
mean arterial pressure and pulse rate. The device is less accurate towards the extremes of BP, more accurate for
systolic than diastolic and the gold standard for BP monitoring is the manual device. In arrhthymias such as AF, the
devices are also inaccurate due to the major fluctuations associated with the individual pulse pressure variations.

Question: 36 of 100
Verapamil:
True / False
prolongs A-V nodal refractoriness Correct

undergoes extensive first-pass hepatic extraction on oral administration Correct

is a useful antiarrhythmic agent in cases of recent myocardial infarction Correct

is contraindicated in Wolff-Parkinson-White syndrome Correct

is useful in the treatment of ventricular tacchyarrhythmias Correct

Verapamil is a calcium antagonist and acts as an antiarrhythmic. Verapamil's antiarrhythmic effects are believed to
be brought about largely by its action on the sinoatrial (SA) and atrioventricular (AV) nodes. Verapamil depresses AV
nodal conduction and prolongs functional refractory periods. Verapamil does not alter the normal atrial action
potential or intraventricular conduction time, but depresses amplitude, velocity of depolarization and conduction in
depressed atrial fibers. Through this action, it interrupts re-entrant pathways and slows the ventricular rate. As a
consequence of the depression of SAN activity paradoxically, using verapamil in the aberrant conduction associated
with WPW may speed up AF and produce VF.
Question: 37 of 100
Peri-operative injury may occur to the:
True / False
optic nerve Correct

ulnar nerve Correct

radial nerve Correct

saphenous nerve Correct

common peroneal nerve Correct

Almost every peripherally located nerve (including all of the above) can be damaged during the peri-operative period.
Neuronal injury is usually temporary and function may gradually return with time (neuropraxia), but occasionally
damage is permanent. The duration spent in an abnormal position is directly correlated to the probability of injury and
possible litigation. The retina and optic nerves are at risk from direct pressure from poorly applied face masks,
surgical instruments, etc. The brachial plexus and its terminal braches are at risk from stretching or direct pressure,
particularly in the lateral position. The lithotomy position is associated with injury to the saphenous, common
peroneal, femoral and obturator nerves.

Question: 38 of 100
The approximate pH of intravenous fluids is as follows:
True / False
Gelofusine is 7.4 Correct

0.9% sodium chloride is 5.0 Correct

Hartmann's solution is 6.0 Correct

5% glucose is 6.8 Correct 4.15

Dextran 70 (in glucose 5%) is 6.5 Correct 4.5

The pH values of intravenous fluids may vary slightly between the different manufacturers, but
approximate pH values (+/- 0.3) are as follows:
 gelofusine is 7.4
 Hartmann's is 6.0
 dextran 40 in 0.9% saline is 6.0.
 sodium chloride 0.9% is 5.0
 dextran 70 in 5% glucose is 4.5
 glucose 5% is 4.15

Question: 39 of 100
Are the following known to release histamine?
True / False
Atracurium Correct

Morphine Correct

Hyoscine Correct
Etomidate Correct

Loratadine Correct

Histamine is released from mast cells following the administration of certain drugs, for example, morphine,
d-tubocurare, atracurium. The release of histamine may cause skin erythema and wheals, bronchospasm,
hypotension, anaphylaxis and possibly cardiac arrest.

 Loratadine is a H1 receptor antagonist (antihistamine).

 Hyoscine is a sedating anticholinergic drug derived from the henbane plant, which only very rarely
triggers histamine release.

Question: 40 of 100
The following are contraindicated in patients receiving Monoamine-oxidase
inhibitors (MAOIs):
True / False
Dopamine Correct

Ephedrine Correct

Pethidine Correct

Dopexamine Correct

Dexamfetamine Correct

Monoamine-oxidase inhibitors (MAOIs) are a class of antidepressant which are prescribed less frequently
than other classes due to their association with dangerous dietary and drug interactions. The inhibition of
the enzyme monoamine-oxidase causes an accumulation of amine neurotransmitters resulting in CNS
and CVS effects. Drugs that should be avoided include: Adrenaline, noradrenaline, ephedrine,
pseudoephedrine, phenylephrine, dopamine, dopexamine, dexamfetamine and pethidine. The metabolism
of sympathomimetics is inhibited which may potentiate their pressor action, and the pressor effect of
tyramine (found in mature cheese, meat and yeast extracts) may also be potentiated. Alcoholic and low
alcohol drinks should also be avoided. Patients taking MAOIs should not be given pethidine as this is
associated with a drug interaction causing CNS and CVS side effects.

The danger of interaction persists for up to 2 weeks after treatment with MAOIs is discontinued, so drug
withdrawal (if appropriate) should be done slowly.

Question: 41 of 100
Diazepam:
True / False
Larger doses are required in renal disease Correct

Is water soluble Correct

Has a longer half-life than nordiazepam (desmethyldiazepam) Correct

Has a similar volume of distribution in adults and paediatric patients Correct

Is highly protein bound Correct


Diazepam is a benzodiazepine that is highly protein bound and insoluble in water. It has a half-life of between 20-70
hours, but is associated with the formation of active metabolites, e.g. nordiazepam (desmethyldiazepam) which has
an even longer half-life (120 hours). The volume of distribution of diazepam in paediatric patients is similar to adult
values but with a greatly prolonged half life. In renal disease diazepam may have a more marked effect owing to
reduced protein binding and a dose reduction is appropriate. Due to impaired phase I metabolism in liver disease,
there is increased sensitivity to diazepam and midazolam.

Question: 42 of 100
A vacuum insulated evaporator (VIE):
True / False
stores oxygen gas at a temperature of minus 150 – 170 degrees Celsius Correct liquid (not gas)

is positioned on a weighing balance to measure the mass of gas Correct mass of the liquid (not gas).

has a safety valve which opens at 700 kPa Correct 1700 kPa (not 700 kPa)

the oxygen is stored at a pressure of 5-10 atmospheres Correct

the vessel is thermally insulated by a vacuum shell Correct

A vacuum insulated evaporator (VIE) is used to store and supply oxygen, and is the most
economical solution in hospitals which have a high demand for oxygen. A VIE is a thermally insulated
container which uses a vacuum shell to insulate the 1500 litres of oxygen contained within. The oxygen is
stored as a liquid (not gas) at a temperature of -150 to -170 degrees Celsius and at a pressure of 5-10
atmospheres. Evaporation of the liquid oxygen requires heat (latent heat of vaporization) and this is taken
from the mass of liquid oxygen within the VIE, thus the low temperature is maintained.

The VIE is positioned on a weighing balance which measures the mass of the liquid (not gas). A resupply
of fresh liquid oxygen is pumped from a tanker into the vessel. A safety valve is located in the VIE that
opens at 1700 kPa (not 700 kPa) and allows oxygen gas to escape into the atmosphere. The pressure in
the VIE increases when there is a reduced demand for oxygen. A pressure regulator maintains the
pressure of the gas entering into the pipeline at 400 kPa.

Question: 43 of 100
Nitrous oxide:
True / False
is more dense than air Correct

supports combustion Correct

is more soluble than nitrogen and oxygen Correct

has a molecular weight of 42 Incorrect answer selected

has a boiling point of 88 degrees Celsius Correct

Nitrous oxide (N2O) is a colourless gas and has been described as having a sweet odour. It is 1.5 times
denser than air. The molecular weight is 44 and it has a boiling point of -88 degrees Celsius. The critical
temperature is 36.5 Celsius and the solubility coefficients are 0.47 (blood gas) and 1.4 (oil/gas).

It is a non-flammable gas which supports combustion, breaking down at high temperatures into oxygen
and nitrogen. N2O expands air filled cavities because it is more soluble than nitrogen. The exact value
quoted ranges from 20 to 40, but the blood/gas solubility coefficient of nitrogen is 0.015, so it is actually 34
times less soluble than nitrous oxide. Thus it passes into the air filled cavity faster than nitrogen can
diffuse out and will quickly expand the volume of a pneumothorax and an air embolism. Remember that
the solubility of a gas depends on the partial pressure, the temperature, the gas and the fluid concerned.

Question: 44 of 100
In immunology the following statements are true:
True / False
in the United Kingdom IgA deficiency may be found in 1 in 500 individuals Incorrect answer selected

chronic liver disease is associated with raised IgA levels, including IgA autoantibodies Incorrect answer
selected

patients with coeliac disease have reduced IgM levels Correct

IgA anti-endomysial antibodies are found in high concentrations in the plasma of patients susceptible to
malignant hyperthermia Correct

Tissue transglutaminase is the main autoantigen responsible for myasthenia gravis Correct

In the United Kingdom, IgA deficiency may be found in as many as 1 in 500 individuals. Patients with IgA
deficiency have a higher than normal incidence of coeliac disease (not IgM deficiency).

Chronic liver disease can result in raised IgA levels, including IgA autoantibodies.

IgA anti-endomysial antibodies and tissue transglutaminase are both related to coeliac disease (not
malignant hyperthermia and myasthenia gravis respectively). IgA anti-endomysial antibodies are highly
diagnostic of coeliac disease, and the main autoantigen responsible for coeliac disease is tissue
transglutaminase.

Question: 45 of 100

Which of the following is/are true regarding diuretics?


True / False
Furosemide directly inhibits the sodium pump in the ascending loop of Henle Incorrect answer selected

Furosemide may reduce renal medullary hypoxia Incorrect answer selected

Loop diuretics are more commonly used than thiazide diuretics in the treatment of hypertension Correct

Mannitol is actively excreted into the renal tubule Incorrect answer selected

Potassium-sparing diuretics are often used as single agents to control hypertension Correct

Furosemide inhibits the Na+/K+/2Cl- co-transporter in the ascending loop of Henle, by binding to the
chloride site. This causes an increased delivery of sodium and water to the distal convoluted tubule, where
an exchange for hydrogen and potassium ions occurs.

The loss of water, sodium, chloride, hydrogen and potassium ions (as well as calcium and magnesium
ions) then occurs. This may lead to hyponatraemia, hypokalaemia and hypochloraemic metabolic
alkalosis.

Loop diuretics are generally used to treat oedema not hypertension.

Potassium sparing diuretics such as amiloride and spironolactone are used in combination with other
diuretics to counteract the loss of potassium. They are rarely used alone.
Mannitol is freely filtered by the glomerulus and is pharmacologically inert as it is neither excreted nor re-
absorbed.

Furosemide may reduce medullary oxygen requirements by diverting blood away from the juxtaglomerular
area to the cortex, but this may have little impact on medullary hypoxia. There is no convincing evidence
that this has renal protective effects.

Question: 46 of 100
Ventilator-associated pneumonia:
True / False
is easily diagnosed Correct

is related to the use of prophylactic antacid therapy Incorrect answer selected

results in an attributable mortality of between 30 - 40% Correct

should not be treated until the causative organism has been identified, in order to avoid the use of broad
spectrum antimicrobial agents Incorrect answer selected

is due to polymicrobial infection in 50% of cases Correct

The diagnosis of ventilator associated pneumonia (VAP) can be extremely difficult to make. Pyrexia,
leukocytosis, purulent bronchial secretions and chest radiograph changes may occur due to non infectious
causes in acute respiratory distress syndrome (ARDS). Bronchial colonisation by uncommon
microorganisms is also common in ITU patients.Invasive methods such as protected bronchial brushes
and bronchoalveolar lavage may improve the sensitivity of microbiological testing.

The prophylactic use of antiulcer agents such as ranitidine has been shown to be associated with gram
negative colonisation of the stomach and upper airways and an increased risk of the development of gram
negative ventilator associated pneumonias.Other risk factors for VAP include: Multiple or frequent
intubations; prolonged mechanical ventilation; aspiration of gastric contents; COPD; and the use of PEEP.

The attributable mortality of VAP (the mortality above that which would be expected by the patient‟s initial
diagnosis) remains unclear. Several studies have found an increased mortality rate of over 50%, whilst
others have found no clear differences in the mortality rate. If a strong index of suspicion exists for VAP,
treatment with broad spectrum antibiotic agents should be initiated rapidly. Treatment is then modified
when microbiological results are known. The use of antibiotics which fail to cover the causative organisms,
or withholding treatment until positive cultures are available, is associated with a higher mortality.

50% of VAPs are due to mixed infections and broad spectrum antibiotics may be required to cover all the
organisms identified. The antibiotic of choice depends upon the rates of isolation of particular pathogens
locally and local resistance patterns. Early consultation with medical microbiologists is advised. Risk
factors for individual organisms include:
 MRSA: Steroids; age > 25 years; COPD; and prior antibiotic use.
 Pseudomonas: COPD; prolonged duration of ventilation; and prior antibiotic use.

Question: 47 of 100
Humidification:
True / False
Reduces mucous plugging and atelectasis Correct
Reduces heat loss Correct

The absolute humidity in the alveoli is 34 g/m3 Correct

The ideal size of a water droplet is about one micron (ìm) Incorrect answer selected

Plate in the ultrasonic nebuliser vibrates at 2 MHz Incorrect answer selected

Medical gases are dry and failure to humidify the inspired gas will cause: drying of the respiratory mucosa
with ciliary dysfunction; mucous thickening with plugging of airways, atelectasis and reduced gas
exchange; and mucosal keratinization and ulceration. Humidification of inspired gases also reduces heat
loss by avoiding the latent heat of vaporization within the trachea and by warming the gases.

The absolute humidity in the proximal trachea is 34 g/m3 where the air is fully saturated at 34 ºC. In the
alveoli the absolute humidity is 43 g/m3, which is fully saturated air at 37 ºC.

The ideal water droplet size is approximately 1 ìm (micron), which is small enough to be deposited in the
alveoli. Droplets <1 ìm tend to be stable and therefore inspired and expired without deposition. Larger
droplets of 20 ìm or greater tend to collect in the breathing system, and 5 ìm droplets tend to be deposited
in the trachea. In the ultrasonic nebuliser water drops onto a plate which vibrates at 2Mz and can achieve
100% saturation.

Question: 48 of 100
Breathing systems:
True / False

A Magill breathing system requires a minimum flow rate of 70 ml per kg per minute during spontaneous
ventilation.
Correct

A Bain breathing system requires a minimum flow rate of 70 ml per kg per min during controlled ventilation.
Correct

During spontaneous ventilation the Lack co-axial system requires a minimum flow rate of 2-3 times the minute
ventilation.
Correct

The Humphrey ADE system only requires 70 ml per kg per min for spontaneous and controlled ventilation.
Incorrect answer selected

Mapleson E and F systems may be used on children under 25 kg.


Correct

The Mapleson classification of breathing systems describes semi-closed systems, excluding the circle
system and non-rebreathing valves. The efficiency of the breathing systems refers to the fresh gas flow
required to prevent rebreathing of alveolar gas. The Mapleson A system is ideally suited for spontaneous
ventilation and requires a minimum fresh gas flow rate of 70 ml/kg/min to prevent rebreathing, which is
equal to the alveolar minute volume.

The Magill and the Lack (a coaxial system) are both classified as Mapleson A systems and they are
inefficient during controlled ventilation.

The Bain breathing system is a coaxial Mapleson D and is efficient during controlled ventilation, requiring
a minimum of 70mls/kg/min to maintain normocapnia. They are very inefficient during spontaneous
ventilation.
The Mapleson E (Ayre‟s T-piece) and F (Jackson Rees modification of the Ayre‟s T-piece) are both
commonly used in paediatrics (< 25Kg) due to its low resistance to breathing. A fresh gas flow of 2 – 3
times the minute volume is required during spontaneous ventilation.

The Humphrey ADE system may change the characteristics of the circuit into a Mapleson A, D or E
dependent on the position of a lever.

Question: 49 of 100
Measurement of the cardiac output by thermodilution:
True / False
10ml of cold fluid is injected through the distal port of the pulmonary artery catheter Correct

a thermistor is located at the tip of the pulmonary artery catheter Correct

a secondary drop in temperature may be seen Incorrect answer selected

cardiac output should be measured during the end-expiratory pause Correct

a curve of the temperature increase against time is displayed Correct

Cardiac output is commonly measured on the ITU using the thermodilution technique, which is easily
performed and allows repeated estimations to be performed. 5 – 10 ml of cold dextrose or saline is
injected through the proximal port (not distal) of the pulmonary artery catheter (PAC). The fluid should be
injected during the end-expiratory pause. The drop in temperature is measured by a thermistor located
proximal to the balloon and is about 4cm from the tip of the PAC. Thus a curve is produced showing the
temperature drop (not increase) against time. A secondary peak is seen when using the dye dilution
techniques, but a secondary drop in temperature is not seen with the thermodilution method. The estimate
of cardiac output is affected by the phase of respiration and it should be measured during the end-
expiratory pause.

Question: 50 of 100

Intraosseous (IO) cannulation:


True / False
Is a technique rarely performed on adults Correct

May be considered when attempts at obtaining venous access are delayed Correct

In the limbs it should be inserted at the fracture site Correct

Is rarely indicated during a paediatric cardiac arrest Correct

Can be inserted in the midline at the level of the tibial tuberosity Incorrect answer selected

First used in the 1930's as an alternative method of gaining vascular access, intraosseous (IO)
cannulation is currently recommended during life threatening situations when vascular access is required
quickly. It can be performed on paediatric and adult patients (though rarely necessary in adults).
Therefore, when attempts at gaining venous access fail or are delayed, intraosseous cannulation should
be considered in patients of any age.

In paediatric cardiac arrest it is the recommended technique for gaining circulatory access and may be
achieved quickly. Many potential anatomical sites for intraosseous cannulation have been described,
including the lower and upper limbs and sternum. The correct site on the proximal tibia is 2 to 3 cm below
the tibial tuberosity on the antero-medial surface (not midline). IO cannulae should not be inserted into
fracture sites, and limbs with proximal fractures should also be avoided.

Question: 5 of 10
Opioid analgesic drugs:
True / False
are more effective systemically than they are intrathecally Correct

have antagonist effects at the opioid receptors Correct

activate receptors linked to ligand gated ion channels Incorrect answer selected

produce the majority of their useful actions at mu receptors Correct

have receptors in the gastrointestinal tract Correct

Opioid receptors exist in the central nervous system and the periphery e.g. the gastrointestinal tract. They are G-protein coupled
receptors (not ligand gated ion channels) and five types have been identified: mu, delta, kappa, sigma and epsilon. They can be
distinguished by their prototype agonists – OP1 (delta), OP2 (kappa) and OP3 (mu).

Opioid drugs are agonists at opioid receptors (not antagonists) and the mu receptor is responsible for the majority of their useful
actions. Intrathecal opioids are more effective than intravenously administered opioids, and the dose should be significantly
reduced to produce the same analgesic effect.

Question: 6 of 10
Dopamine:
True / False
At a dose of 5-10mcg/kg/min, its action is mainly at the alpha adrenoceptors. Correct

When given as an infusion dopamine reduces the progression to renal failure Correct

Infusions may increase splanchnic blood flow. Correct

Has antiemetic properties Correct

Crosses the blood brain barrier Correct

Dopamine acts predominately at the dopamine receptors at low dose (1-5mcg/kg/min), at beta
adrenoceptors at intermediate doses (5-10mcg/kg/min) and at alpha receptors at high doses (10-
15mcg/kg/min). Its use doesn't affect the progression to renal failure, although urinary output is often
increased by its use. It has marked emetogenic effects (not antiemetic) by the action at the chemoreceptor
trigger zone, which is outside the blood brain barrier.

However, L-dopa does cross the blood brain barrier and is therefore, used in the treatment of Parkinson's
disease.

Question: 7 of 10

Insulin:
True / False

interacts with the nuclear membrane Correct

causes an increased glucose-protein transport on the endoplasmic reticulum Correct

acts via a similar mechanism to steroid receptors Correct

can be detected in the lymph Correct


is synthesised in the alpha cells of islets of Langerhans Correct

Insulin is synthesised in the beta cells of the islets of Langerhans, which interacts with cell surface
receptors (not the nuclear membrane and so is not like steroids). Insulin binding to its receptor results in
receptor autophosphorylation on tyrosine residues and the tyrosine phosphorylation of insulin receptor
substrates (IRS-1, IRS-2 and IRS-3) by the insulin receptor tyrosine kinase.

Question: 8 of 10
The following drugs need dose adjustment in renal failure:
True / False

Aciclovir Correct

Carbamazepine Incorrect answer selected

Digoxin Correct

Erythromycin Correct

Vancomycin Correct

Aciclovir may cause nephrotoxicity by intratubular aciclovir crystalluria, and the dose should be reduced in
renal failure.

Carbamazepine is approximately 80% protein bound and is metabolised in the liver, but is renally
excreted. Renal impairment has little effect on the dose, so dose adjustment is not necessary, but the
manufacturer does advise caution.

Digoxin and vancomycin both require dose adjustment in patients with renal failure.

In severe renal impairment the dose of erythromycin should be limited to 1.5 g per day because of the risk
of ototoxicity.

Question: 9 of 10
Histamine type 2 (H2) receptor antagonists:
True / False

reduce the volume of gastric secretions Correct

cause vasoodilation Correct

increase pancreatic secretion Correct

decrease gastric pH Correct

may cause sedation Correct

Histamine type 2 (H2) receptor antagonists reduce acid secretion (thus lowers the volume) and causes an
increase in pH (not decrease). Pancreatic secretion is also increased. Vasodilatation is associated with
histamine type 1 (H1) and H2 receptor agonists (not antagonists). Sedation is more a feature of the older
H1 antagonists (not H2 antagonists) which can cross the blood brain barrier.

Question: 10 of 10

Concerning the pharmacology of coagulation:


True / False

The effects of heparin are monitored using the International Normalized Ratio Correct
Carbamazepine reduces the effect of Warfarin Correct

Aprotinin is fibrinolytic Correct

Alteplase is used in thrombolysis Correct

Factor VIIa (recombinant) is an anticoagulant Correct


Heparin is an anticoagulant and its effects are monitored by measuring the activated partial
thromboplastin time (APTT), although thrombin and clotting times are also prolonged (not INR). It
accelerates the action of antithrombin III, which inhibits activated factors XII, XI, X, IX and thrombin.
Warfarin is an oral anticoagulant and it inhibits the synthesis of vitamin K dependant factors II, VII, IX and
X. Hepatic enzyme inducing agents, e.g. carbamazepine and phenobarbitone reduce its effect. Enzyme
inhibitors such as valproate enhance the effect of warfarin. If the enzyme-inducing drug is withdrawn
without reducing the dose of warfarin, haemorrhage may occur. Aprotinin is an enzyme inhibitor acting on
plasmin and kallikrein and is classed as an antifibrinolytic (not fibrinolytic), thus inhibiting fibrinolysis. It is
indicated in patients at high risk of blood loss. Alteplase (tissue type plasminogen activator) is a
thrombolytic and acts by activating plasminogen to form plasmin, which degrades fibrin and so breaks up
thrombi. It is used in the treatment of myocardial infarction and life-threatening venous thrombosis. Factor
VIIa (recombinant) is a purified coagulation factor used to treat patients with inhibitors to factors VIII and
IX. It has been used successfully in patients with serious trauma in whom haemorrhage has been difficult
to control surgically. Thus it can be classed as a procoagulant (not anticoagulant).
Question
The following may be safely used in a patient susceptible to malignant
hyperthermia:
True / False

Glycopyrrolate Correct

Atropine Correct

Neostigmine Correct

Droperidol Correct

Metoclopramide Correct

Known triggering agents for malignant hyperthermia (MH) include all of the volatile agents and
suxamethonium. „Safe drugs‟ have been evaluated in the laboratory and safely used in MH susceptible
patients. These include all intravenous induction agents, all benzodiazepines, all non-depolarising muscle
relaxants, all local anaesthetic agents (including those with vasoconstrictors), all analgesics and opioids.
However, not all drugs used during anaesthesia have been thoroughly tested, but expert advice is
available from the Leeds MH Investigation Unit.
Question
Ephedrine:
True / False

May increase fetal acidosis Correct

Is the preferred vasoconstrictor in obstetric anaesthesia Incorrect answer selected

Is positively inotropic and chronotropic Correct

Is used to maintain uteroplacental blood flow Correct

Is used to reduce intervillous blood flow Correct

Ephedrine is used in obstetric anaesthesia because it has vasoconstrictor, chronotropic and inotropic
properties. It normally maintains uteroplacental and intervillous blood flow, which preserves the foetal
blood supply and prevents deterioration of the fetal acid base status. However, new work suggests this
may not always be the case. Therefore, alpha agonists such as phenylepherine are now the first choice in
obstetric anaesthesia.
Question
At the volar aspect of the wrist:
True / False

The median nerve is lateral to palmaris longis Incorrect answer selected

The ulnar nerve is medial to the ulnar artery Correct

The median nerve is medial to flexor carpi radialis Correct

The ulnar nerve is medial to flexor carpi ulnaris Incorrect answer selected

The radial artery is a larger vessel than the ulnar artery Incorrect answer selected
The usual relationship between the nerves, arteries and tendons (radial side to ulnar side) at the volar
aspect of the wrist is as follows: Radial artery, flexor carpi radialis, median nerve, palmaris longis, ulnar
artery, ulnar nerve and flexor carpi ulnaris. The ulnar artery is the larger vessel.
Question
When analysing data derived from pain assessment scales which of the
following is/are true?
True / False

A pain relief score may give more useful information than a change in pain score Incorrect answer selected

A time series of numerical rating scores are best analysed using the Mann-Whitney test Correct

A time series of visual analogue scores are best analysed using an 'area under the curve' approach with t-tests
Incorrect answer selected

The data from a three-level verbal rating scale may be compared with the χ2 test Incorrect answer selected

Two groups of visual analogue scores may be compared using thet-test Incorrect answer selected

Verbal rating scales (VRS) and numerical rating scales (NRS) generate discontinuous data that are
unsuitable for parametric tests of statistical significance and thus non-parametric techniques must be
used.

When the VRS is confined to only three levels, data can be summarised in contingency tables and either
the χ2 test or exact tests used. Where VRS is divided into several levels or NRS used, the Mann-Whitney
test or Wilcoxon rank sum test is appropriate.

Visual analogue scales (VAS) yield continuous data andt-tests can be used as long as less than 25% of
the data are at extreme ends of the range. If there are doubts about the validity of a t-test, non-parametric
tests can be used. VAS data may be analysed using standard deviation and standard error.

A time series of numerical rating scores are best analysed using some form of analysis of variance for
repeated measures or even area under the curve. Measuring the area under the curve gives a summary
measure for each patient that can be analysed by a single test.

The Mann-Whitney test only compares two sets of data and cannot be used for multiple testing.
Carbon Monoxide (CO):
True / False

Induces cytochrome oxidase Correct

Causes a left shift in the oxygen-haemoglobin dissociation curve Correct

Clearance is greater in females Correct

Is negatively inotropic Incorrect answer selected


May exacerbate myocardial ischaemia Correct

Haemoglobin (Hb) has 250 times more affinity for carbon monoxide than for oxygen, which reduces the
total amount of Hb available for oxygen transport. CO shifts the oxygen-haemoglobin dissociation curve to
the left and down, reducing the ability of Hb to release oxygen. CO inhibits cytochrome oxidase, which
reduces mitochondrial ATP formation and worsens tissue hypoxia. Clearance is decreased in men and
during sleep. CO is negatively inotropic. Carboxyhaemoglobin (COHb) levels of 4.5 to 6% reduce the
onset time of exercise induced angina and increase the incidence of ventricular dysfunction and
dysrhythmias. Myocardial ischaemia itself promotes the formation of carboxyhaemoglobin, which further
reduces oxygen delivery to the ischaemic myocardium.
Bumetanide:
True / False

Causes a metabolic alkalosis Correct

Increases oxygen consumption in the nephron Correct

Is filtered into the tubular lumen Correct

May reduce ventricular preload Correct

Causes hypermagnesaemia Correct

Bumetanide is a loop diuretic which is secreted into the tubular lumen by an organic acid transporter and
inhibits the Na+K+2Cl- co-transporter in the ascending loop of Henle, leading to an increased delivery of
sodium and water to the distal convoluted tubule.

Oxygen consumption in the nephron is decreased by loop diuretics and in the loop of Henle this may be
reduced to basal levels.

Loop diuretics are associated with


 hypokalaemia
 hyponatraemia
 hypomagnesaemia
 metabolic alkalosise.

They can also reduce ventricular preload by causing vasodilatation.

Hyperglycaemia may follow the administration of:


True / False

Gelofusine Correct

Blood Correct

Hartmann's solution Correct

0.45% saline in 5% dextrose Correct

0.45% saline in 2.5% dextrose Correct

Blood has been shown to have high glucose content and when transfused in large volumes will cause
hyperglycaemia. Although 0.45% saline in 2.5% dextrose does contain glucose it has not been shown to
cause intraoperative hyperglycaemia. Gelofusine and Hartmann's solution do not contain glucose but the
lactate in Hartmann's solution may be metabolised to glucose and hyperglycaemia has been
demonstrated in diabetic patients.
Nitroprusside:
True / False

May induce a metabolic acidosis Correct

Is an indirectly acting nitric oxide donor Incorrect answer selected

May cause a tachycardia Correct

May increase the alveolar-arterial oxygen difference Correct

Inhibits hypoxic pulmonary vasoconstriction Correct

Sodium nitroprusside is a vasodilator and is the only directly acting nitric oxide (NO) donor in common
use. NO is spontaneously released rather than being released following an enzymic or chemical reaction
(as in indirectly acting NO donors). It has 5 cyanide moieties for every nitric oxide moiety, so cyanide
toxicity and a metabolic acidosis may occur. Monitoring cyanide concentration and base excess is
advised. It is equipotent as an arterial and venous dilator, and the reduction in arterial pressure may lead
to a reflex tachycardia. It reverses hypoxic pulmonary vasoconstriction and worsens the alveolar-arterial
oxygen difference.

Remifentanil:
True / False

Acts at most opioid receptors Correct

Does not release histamine Correct

Plasma pseudo-cholinesterase deficiency prolongs its clearance Correct

Is an anilidopiperidine derivative Correct

Has a peak effect at 30 seconds Correct

Remifentanil is a synthetic anilidopiperidine derivative, which is a pure μ agonist. The peak effect is
between 1 and 3 minutes. It is metabolised by non-specific tissue esterases, which is not affected by
genetics, renal failure, hepatic failure or age. It has a context-sensitive half time of 3 to 5 minutes and
does not accumulate in the tissues. It is not associated with histamine release.

Carbonic anhydrase:
True / False

Contains iron Correct

Has 4 isoenzymes Correct

Is present in the plasma Correct

Is inhibited by spironolactone Correct

Isoenzyme IV is found in the brush border of the proximal convoluted tubule Correct

Carbonic anhydrase is a zinc containing enzyme which is found in erythrocytes (not plasma), the nephron,
pulmonary endothelium, gut, pancreas, cardiac and skeletal muscle. Seven isoenzymes have been
identified to date. Isoenzyme IV is found in the in the brush border of the proximal convoluted tubule and
isoenzyme II is found within the luminal cells. It is inhibited by Acetazolamide, which is used as
prophylaxis against mountain sickness and in the management of glaucoma. Spironolactone is an
aldosterone antagonist and is classed as a potassium sparing diuretic.

The following are indications for urgent endotracheal intubation:


True / False

Glasgow coma score of 9 Correct


LeFort III facial fracture Correct

Facial surgical emphysema Correct

Glasgow coma motor score of 2 Correct

LeFort I facial fracture Correct

Simple airway opening manoeuvres, the delivery of high flow oxygen and use of airway adjuncts (with
cervical spine protection) following the ABC approach to trauma forms the basis of management.

The indications for urgent endotracheal intubation include a GCS of 8 or less and an extensor response to
pain (motor score of 2).

In facial trauma, endotracheal intubation is never undertaken lightly. In the absence of neurological injury,
isolated LeFort fractures (I to III) rarely require urgent intubation, unless complicated by pharyngeal
haemorrhage or oedema.

Inulin:
True / False

has a molecular weight of 5000 This is the correct answer

is an endogenous molecule This is the correct answer

is uncharged This is the correct answer

does not cross the cell membrane This is the correct answer

its elimination is exponential This is the correct answer


Inulin is a polyfructose (from Jewish artichokes) and has a spherical configuration with a molecular weight
of 5000. It freely filters through the glomerular barrier and is uncharged and not bound to plasma proteins.
Inulin freely crossed most capillaries but does not traverse the cell membrane (distribution volume is the
ECF). As an exogenous molecule inulin is not synthesised or broken down in the body and all ultrafiltrated
inulin molecules pass into the urine as they are neither secreted nor reabsorbed. The elimination of inulin
is exponential, i.e. the fraction (k) of the remaining amount in the body that disappears per unit time is
constant.

The following increase in concentration during pregnancy:


True / False

total cortisol Correct

growth hormone Incorrect answer selected

angiotensin Correct

aldosterone Correct

free plasma thyroxin Incorrect answer selected

The pituitary gland increases the production of prolactin and adrenocorticotrophin (ACTH) but the
production of growth hormone is reduced (not increased). Plasma concentrations of cortisol, aldosterone,
rennin and angiotensin rise. There is an increase in the production thyroid hormone, but an increase in
production of thyroid binding globulin means that the free plasma concentration of thyroid hormones
(thyroxin) remains unchanged (not increased).

In anaesthetic gas cylinders:


True / False
Size E cylinders are usually fitted to the anaesthetic machine Correct

Size E oxygen cylinders contain 1360 litres Correct

Size E nitrous oxide cylinders contain 1800 litres Correct

Entonox cylinders only available in size J Correct

Size D oxygen cylinders contain 360 litres Correct

The ideal size for a cylinder fitted to the anaesthetic machine is size E. The other cylinder sizes that are
asked about are discussed as below:
 Oxygen cylinders range from size C to size J. Size C oxygen cylinders contains 170 litres, and as
the size of the cylinder increases the volume of oxygen inside doubles each time. Thus a size D
contains 340 litres (not 360 litres), a size E contains 680 litres (not 1360), and so on.
 Nitrous oxide cylinders are available in sizes C to G and similarly the volume doubles as the size of
the cylinder increases. Size C cylinders contain 450 litres of nitrous oxide; therefore size E
cylinders must contain 1800 litres.
 Entonox is available in a wide range of cylinder sizes including D, G and J.

In muscular dystrophy:
True / False

plasma creatine kinase concentrations will be low Correct

a pre-operative 12 lead ECG is required Correct

hyperkalaemia and myoglobinuria may occur post-operatively Correct

these patients quickly develop tolerance to opioid analgesics Correct

post-operative chest infections are common Correct

Muscular dystrophies are rare hereditary disorders of muscle. Progressive destruction of skeletal and
cardiac muscle occurs by a mechanism thought to involve abnormal muscle membrane function. Plasma
creatine kinase level may be increased. Patients tend to present with limb contractures. They have weak
respiratory muscles which impairs ventilation and sputum clearance. Pre-existing and post-operative chest
infections are common and they are especially sensitive to opioids and other respiratory depressant
drugs. Severe hyperkalaemia and myoglobinuria may occur following prolonged exposure to volatile
anaesthetic agents, therefore regional techniques are preferred. As arrhythmias and cardiac failure may
occur due to myocardial involvement, an ECG and echocardiogram will provide valuable pre-operative
information on cardiac function.
Pulmonary fibrosis is a side effect of:
True / False

nalidixic acid Correct

vincristine Correct

sulphasalazine Correct

corticosteroids Correct

penicillamine Correct
Pulmonary fibrosis is a side effect of the following drugs: Gold, penicillamine and methotrexate which are used in
rheumatology (not nalidixic acid); Cytotoxic drugs, e.g bleomycin and cyclophosphamide (not vincristine);
sulphasalazine, amiodarone and nitrofurantoin also cause pulmonary fibrosis. However, corticosteroids do not and
are sometimes used to treat pulmonary fibrosis.

Pleural calcification occurs in:


True / False
tuberculosis Correct

bagassosis Correct

siderosis Correct

chronic empyema Correct

asbestosis Correct
Pleural calcification is seen in tuberculosis, chronic empyema and asbestosis, but not in bagassosis nor
siderosis.

A 55-year-old patient with a long history of rheumatoid arthritis is admitted for a laparoscopic
cholecystectomy.
Which of the following statements are true?
True / False

A lateral cervical spine x ray is required Correct

A restrictive pattern on lung function tests is a feature of lung involvement Correct

Anaemia is a common pre-operative finding Correct

Approximately 10% of patients have cervical spine instability Correct

Stridor and airway obstruction may occur due to cricoarytenoid involvement Correct

Rheumatoid arthritis is a systemic connective tissue disease, which presents as a symmetrical arthropathy
involving any joint (except the terminal interphalangeal joints).

Lung involvement usually causes fibrosis and hence a restrictive pattern on lung function testing.

Atlanto-axial ligament laxity together with odontoid peg erosion may result in cervical subluxation and
possible cord compression.

25% of rheumatoid arthritis sufferers have cervical instability but only 7% have clinical signs. Cervical
spine x rays are required and a gap of more than 3 mm between the odontoid peg and the posterior arch
of the axis is diagnostic of subluxation.

Cricoarytenoid involvement may cause hoarseness, stridor and airway obstruction. A normochromic
normocytic anaemia plus anaemia from chronic gastrointestinal loss (NSAIDs) are often found on a pre-
operative full blood count.

Bone marrow suppression is a side effect of gold therapy and penicillamine can cause thrombocytopenia.

Pancytopenia associated with hepatosplenomegaly is termed Felty's syndrome.


The following are primary indications for insertion of an arterial cannula:
True / False

beat-to-beat assessment of blood pressure Correct

regular urea and electrolyte monitoring Correct

multiple arterial blood sampling Correct

to monitor the oscillation on the arterial trace in order to determine fluid status Correct

to measure heart rate variability Correct


Beat-to-beat assessment of blood pressure and multiple arterial blood sampling are the two primary
indications for insertion of arterial cannulae. Urea and electrolyte samples may be taken via a venous
cannula or by venepuncture.

Monitoring the oscillation or “swing” of an arterial line does provide information on the volaemic status of
the patient but there are more accurate methods available. Heart rate variability is primarily measured
using electrocardiograph monitoring.
The following are associated with resistance to non-depolarising
neuromuscular blockade:
True / False

Hypothermia Correct

Myasthenia Gravis Correct

Hemiplegia Correct

Burns Correct

Antiepileptic drugs Correct

Resistance to non-depolarising muscle relaxants is associated with: Burns, antiepileptic drugs,


intracranial lesions, and hemiplegia (when neuromuscular function is monitored on the affected
side). Long-term phenytoin therapy shortens the duration of action of long-acting muscle
relaxants by 50%. Hypothermia is associated with prolongation (not resistance) of neuromuscular
blockade and patients with myasthenia gravis are extremely sensitive to non-depolarising
neuromuscular blockade.
Are the following true about blood coagulation?
True / False

Addition of vitamin K to freshly drawn blood delays clotting Correct

Heparin inhibits blood coagulation through its interference with vitamin K metabolism in the liver Correct

Platelets are essential for blood clot Correct

Thrombin converts fibrinogen to fibrin Correct

Vitamin K is required for the production of factors II, VII, IX and X Correct

Regarding blood coagulation:

Heparin increases the activity of anti-thrombin III which inhibits the clotting cascade.

Vitamin K is used in liver to produce clotting factors and on its own has no anti-coagulation effect.

Thrombin is important in the clotting cascade by releasing fibrin from fibrinogen.

Platelets are essential for clotting factors II, VII, IX and X and are produced in liver with vitamin K.
The parasympathetic nervous system:
True / False

The facial nerve supplies parasympathetic secretomotor fibres to the parotid salivary gland Correct

The glossopharyngeal nerve leaves the skull through the stylomastoid foramen Correct

The vagus nerve enters the abdomen with the aorta Correct

The sacral connector cells are found in the gray matter of the second, third and fourth sacral segments of the cord
Correct
The postganglionic fibres are relatively short and are nonmyelinated Correct

The connector cells of the parasympathetic part of the autonomic nervous system (preganglionic efferent
fibres) are located in the brain and sacral segments of the spinal cord. Those in the brain form part of the
nuclei of origin of cranial nerves III, VII, IX and X and the axons emerge from the brain contained in the
corresponding cranial nerves. The sacral connector cells are found in the gray matter of the second, third
and fourth sacral segments of the cord. The myelinated axons leave the cord in the anterior nerve roots of
the corresponding spinal nerves and finally from the pelvic splanchnic nerves. The cranial preganglionic
(efferent) fibres relay in the ciliary, pterygopalatine, submandibular and otic ganglia, whereas those in the
pelvic splanchnic nerves relay in ganglia in the pelvic plexuses. Classically the postganglionic fibres are
nonmyelinated and are relatively short in length compared with those of the sympathetic system. The
facial nerve (VII) emerges from the skull through the stylomastoid foramen. The parasympathetic
secretomotor fibres supply the submandibular and sublingual salivary glands, the lacrimal gland and the
glands of the nose and palate. The glossopharyngeal nerve (IX) leaves the skull by passing through the
jugular foramen (not stylomastoid foramen) and supplies the parotid salivary gland with parasympathetic
secretomotor fibres (not facial nerve). The vagus nerve (X) also leaves the skull through the jugular
foramen. Each vagus nerve then descends through the neck alongside the carotid arteries and internal
jugular veins (within the carotid sheath), traverses the chest before finally piercing the diaphragm with the
oesophagus (not aorta) to terminate within the abdomen.
Which of the following is/are true regarding labetalol?
True / False

Causes bronchodilation Correct

Decreases bile secretion Correct

Has a half life of two hours Correct

Has alpha blocking action Correct

Is 70% protein bound Correct

Labetalol is a combined beta and alpha adrenergic receptor antagonist with a ratio of activity between 2:1
and 5:1 respectively.

It is selective for alpha-1 receptors but is non-selective for beta receptors.

It is used to treat severe hypertension and pre-eclampsia and in hypotensive anaesthesia.

Labetalol has a half life of approximately four hours (not two) and is approximately 50% protein bound.

It has been shown to cross the placental barrier, but not the blood brain barrier. It is metabolised in the
liver and excreted in the urine and faeces.

Oral administration of the drug undergoes extensive first-pass metabolism.

Severe hepatocellular damage has been reported after both short and long term use and the reduction of
bile secretion may rarely lead to jaundice.
Sucralfate:
True / False

Is a simple compound formed from sulphated sucrose Correct

The molecules polymerise below pH 2 Correct

Is an antacid Correct

Has very few side effects Correct


Has an antibacterial action against Helicobacter pylori Correct

Sucralfate is classed as a mucosal strengthener as it increases mucosal resistance to acid-pepsin attack.


It is a complex (not simple) substance formed from sulphated sucrose and aluminium hydroxide. The
molecules polymerise below pH4 (not pH 2) to give a sticky gel that strongly adheres to ulcer craters. The
protective action of the gel allows the development of the normal pH gradient caused by the secretion of
bicarbonate ions normally present in the mucous layer. Sucralfate is not an antacid and it has very few
side effects. Bismuth chelate has an antibacterial action against Helicobacter pylori.
The following drugs do not inhibit cyclo-oxygenase:
True / False

Paracetamol Correct

Misoprostol Correct

Ethacrynic acid Correct

Azapropazone Correct

Phenylbutazone Correct

Note that the question asks which drugs DO NOT inhibit cyclo-oxygenase (COX). Cyclo-oxygenase
catalyses the formation of endoperoxidases from arachidonic acid. Endoperoxidases are subsequently
converted into prostaglandins, thromboxane and prostacycline. The enzyme is inhibited by azapropazone
and phenylbutazone, which are non-steroidal anti-inflammatory drugs (NSAIDs). Paracetamol is believed
to inhibit central cyclo-oxygenase, possibly COX 3. Misoprostol is a synthetic prostaglandin analogue and
does not inhibit cyclo-oxygenase. Ethacrynic acid is an unsaturated ketone derivative of an aryloxyacetic
acid and is classed as a loop diuretic. It inhibits the reabsorption of sodium and chloride in the medullary
portion of the ascending limb of Henle‟s loop.
Tetracyclines:
True / False

Inhibit transpeptidation Correct

The simultaneous administration of aluminium hydroxide impedes its absorption Correct

Bind to the 50S subunit Correct

Do not penetrate macrophages Correct

Are safe in pregnancy Correct

Tetracyclines are broad-spectrum antibiotics that penetrate macrophages and are often the drugs of
choice for treating infections due to intracellular organisms. Sensitive organisms accumulate the drug by
both active and passive transport, e.g. chlamydia and rickettsia. Tetracyclines bind to the 30S subunit (not
50S) of the bacterial ribosome and inhibit the binding of the aminoacyl-transfer-RNA. Traspeptidation is
inhibited by chloramphenicol. Deposition of tetracyclines in growing bone and teeth due to calcium binding
causes dental discolouration (and dental hypoplasia) and they should be avoided in children under the
age of 12 years and in pregnant or lactating mothers. Aluminium hydroxide if given simultaneously with
oral tetracyclines may form complexes that can impede its absorption. Tetracyclines may increase the
muscle weakness in myasthenia gravis and should be used with caution in patients with hepatic
impairment.
Do the following drugs act by blocking sodium channels?
True / False

Carbamazepine Correct

Diazepam Correct

Ethosuximide Correct
Minoxidil Correct

Suxamethonium Correct

Carbamazepine is an anticonvulsant and blocks neuronal sodium channels reducing the sodium ion flux
until it is insufficient to evoke an action potential.

Suxamethonium is a depolarising neuromuscular blocking drug that causes inactivation of the voltage
sensitive sodium channels in the muscle fibre membrane.

Diazepam is a benzodiazepine, which increases the chloride ion conductance of the neuronal membrane.

Minoxidil is a potent vasodilator that opens ATP-sensitive potassium channels in vascular smooth muscle,
causing hyperpolarisation and relaxation.

Ethosuximide may act by reducing the calcium ion flux through T-type calcium channels in thalamic
neurones and is used in the treatment of absences and myoclonic seizures.

Review Question
Regarding omeprazole, which of the following is/are true?
True / False

Has an elimination half life of 120 minutes Correct

Is a reversible enzyme inhibitor Correct

Is inactive at neutral pH Correct

May inhibit the metabolism of diazepam Correct

Reacts with hydroxyl groups in the hydrogen ion/potassium ion ATPase Correct

Omeprazole is a proton pump inhibitor (PPI) that is inactive at neutral pH.

In an acid environment it rearranges into two types of reactive molecule that react with sulphydryl groups
in the hydrogen ion/potassium ion ATPase (not hydroxyl groups).

The enzyme is irreversibly inhibited and thus acid secretion resumes only after new enzyme is
synthesised.

Omeprazole has an elimination half life of 40 minutes.

Proton pump inhibitors are associated with many drug interactions that include enhancing the effect of
phenytoin and warfarin, and the possible inhibition of diazepam metabolism.

Review Question
Which of the following inhibit cyclo-oxygenase?
True / False

Rofecoxib Correct

Aspirin Correct

Clopidogrel Correct

Losartan Correct
Domperidone Correct

Aspirin like most NSAIDs act through inhibition of cyclo-oxygenase. Rofecoxib is a selective COX-II
inhibitor and is used specifically as it is not associated with the inhibition of COX-I responsible for the
production of mucosal protection in the stomach. Clopidogrel is an ADP receptor antagonist and
responsible for inhibtion of platelet aggregation through this route. Losartan is an Angiotensin antagonist
and Domperidone is a dopamine antagonist.

Review Question
Regarding cimetidine, which of the following is/are true?
True / False

Has anti-androgenic actions Correct

Is inactive at neutral pH Correct

May reduce the metabolism of theophylline Correct

Reduces both vagal and gastrin induced acid secretion Correct

Reduces intracellular cyclic-AMP in the parietal cells Correct

Cimetidine is a histamine H2-receptor antagonist that blocks the action of histamine on the parietal cells.

Parietal cells secrete hydrochloric acid into the stomach lumen, which is stimulated by acetylcholine and
gastrin. Although parietal cells possess muscarinic (M1) and gastrin receptors, both acetylcholine and
gastrin mainly stimulate acid secretion indirectly, by releasing histamine from paracrine cells located close
to the parietal cells.

The histamine then acts locally on the parietal cells, where activation of H2-receptors results in an
increase in intracellular cyclic-AMP and the secretion of acid.
Thus cimetidine reduces both intracellular cyclic-AMP levels and acid secretion. Since acetylcholine and
gastrin act indirectly by releasing histamine, the effects on acid secretion of both gastrin and vagal
stimulation are reduced by H2-receptor antagonists.
It is rapidly absorbed orally and it relieves the pain of the peptic ulcer. It binds to cytochrome P-450 and
may reduce the hepatic metabolism of theophylline, phenytoin and warfarin. It also has slight anti-
androgenic actions and may rarely cause gynaecomastia.
Omeprazole is inactive at neutral pH (not cimetidine).

Review Question
Are the following antiplatelet agents?
True / False

Aspirin Correct

Clopidogrel Correct

Heparin Correct

Nitric oxide Correct

Warfarin Correct

Heparin acts through antagonism of a number of factors in the coagulation cascade, as does warfarin
through inhibition of the vitamin K derived factors.
Nitric oxide is the key factor produced by the endothelium resulting in endothelial dependent dilatation as
well as inhibiting platelet aggregation.

Clopidogrel inhibits platelet aggregation through inhibition of the adenosine diphosphate (ADP) receptors
on the platelets.

Aspirin has a similar function but mediated through inhibition of cyclo-oxygenase.

In high dosage heparin may have an antiplatelet effect, but it is also associated with heparin-induced
thrombocytopenia (HIT). HIT is due to heparin combining with heparin-binding protein in platelets (PF4).

This antigenic complex induces the formation of IgG, which binds platelets causing clumping. If severe
enough consumptive thrombocytopenia and thrombosis follow.

Review Question
The following drugs are safe in porphyria:
True / False

Morphine Correct

Thiopental Correct

Digoxin Correct

Clonidine Correct

Amiodarone Correct

The acute porphyrias are hereditary disorders of haem biosynthesis and caution must be exercised when
prescribing. Many drugs can induce acute porphyric crises and it is essential to ensure that a drug is safe
before administering it. However, the treatment of life threatening conditions should not be delayed in
patients with acute porphyria. When an alternative drug is unavailable, urinary porphobilinogen excretion
should be regularly measured and if it increases or symptoms occur, the drug should be withdrawn. In
severe acute porphyric crises, haem arginate should be given by intravenous infusion as haem
replacement. Fentanyl, morphine, diamorphine, codeine, dihydrocodeine, pethidine and buprenorphine
are thought to be safe. Barbiturates (thiopental and methohexital), etomidate, steroids, alcohol, Clonidine,
amiodarone and the progestogens are all considered unsafe. For a full list of drugs available on the UK
market that have been classified as unsafe you should refer to the BNF or the Welsh Medicines
Information Centre at the University Hospital Wales in Cardiff.

Review Question
The blood supply of the lungs:
True / False

The bronchial arteries are branches of the pulmonary arteries Correct

The alveoli receive oxygenated blood from the terminal branches of the pulmonary arteries Correct

There are two pulmonary veins Correct

The pulmonary veins follow the intersegmental connective tissue septa to the lung root Correct

The visceral pleura receives its blood supply from branches of the descending aorta Correct

The bronchial arteries are branches of the descending aorta (not pulmonary arteries) and supply the
bronchi, the lung connective tissue and the visceral pleura. The bronchial veins communicate with the
pulmonary veins and drain into the azygos and hemiazygos veins. The terminal branches of the
pulmonary arteries deliver deoxygenated blood to the alveoli (not oxygenated). Following oxygenation the
blood leaves the alveolar capillaries and drains into the tributaries of the pulmonary veins, which follow the
intersegmental connective tissue septa to the lung root, thus there are a total of four (not two).

Review Question
May the following clinical conditions prolong neuromuscular blockade?
True / False

Hypercalcaemia Correct

Hypermagnesaemia Correct

Hypokalaemia Correct

Metabolic alkalosis Correct

Respiratory acidosis Correct

The clinical conditions which are known to prolong or potentiate non-depolarising neuromuscular blockade
include
 Hypokalaemia
 Hypocalcaemia
 Hypermagnesaemia
 Metabolic alkalosis
 Respiratory acidosis and
 Hypothermia.

Numerous drugs are also associated with prolonged neuromuscular blockade


 Antibiotics such as streptomycin, polymyxin and neomycin
 Cocaine, procaine and lidocaine and
 Lithium due to its hypokalaemic effect.

The features of hypercalcaemia on skeletal muscles is to cause muscle weakness, but this is not
commonly associated with a prolongation of the neuromuscular block.

Conversely the features of hypocalcaemia include muscle cramps, spasm and tetany and these patients
may have a shorter block, but this is rarely seen in practice.

Review Question
Is it true/false that the following include the side effects of total parenteral
nutrition?
True / False

Fatty acid deficiency Correct

Hypercarbia Correct

Hyperglycaemia Correct

Hypoglycaemia Correct

Metabolic acidosis Correct

The side effects of total parenteral nutrition (TPN) are numerous and include catheter-related sepsis and
metabolic abnormalities resulting from the administered nutrients.
Fatty acid deficiency may develop during prolonged TPN, though administering 3% of the total caloric
input as linoleic acid prevents or corrects this deficiency.

Hyperchloraemic metabolic acidosis may occur because of the liberation of hydrochloric acid during the
metabolism of amino acids in the TPN.

Hypercarbia occurs from the increased production of carbon dioxide resulting from the metabolism of large
amounts of glucose. A requirement for ventilatory support or weaning difficulties may subsequently occur.

Hyperglycaemia is a potential problem until endogenous insulin production increases, requiring frequent
glucose monitoring.

Hypovolaemia due to an osmotic diuresis and a non-ketotic hyperosmolar hyperglycaemic coma are both
potential complications of TPN, which may necessitate the addition of insulin to the TPN solutions.

Accidental or sudden discontinuation of the TPN infusion may cause hypoglycaemia.

The pancreatic insulin response may persist despite discontinuing the TPN resulting in a high plasma
insulin concentration. Consequently intravenous glucose administration may be required, or alternatively a
gradual discontinuation of the TPN over 60 to 90 minutes.

Review Question
Nitroprusside:
True / False

decreases platelet aggregation Correct

may reduce the arterial PO2 Correct

may cause a coronary steal syndrome Correct

causes an increased release of rennin Correct

may cause an increase in the mixed venous PO 2 Correct


Nitroprusside is an organic nitrate that produces nitric oxide, which activates the enzyme guanylate cyclase and
increases the concentrations of cyclic guanosine monophosphate that relaxes vascular smooth muscle. The
metabolism of nitroprusside begins with the transfer of an electron from the iron of oxyhaemoglobin to nitroprusside,
yelding methaemoglobin and an unstable nitroprusside radical. This unstable radical breaks down to release all five
cyanide ions forming cyanmethaemoglobin and thiocyanate. The appearance of tachyphylaxis with a metabolic
acidosis and an elevated mixed venous PO2 is suggestive of cyanide toxicity. The increased mixed venous PO2 is
due to paralysis of cytochrome oxidase and inability of the tissues to utilise oxygen and the metabolic acidosis is due
to tissue anaerobic metabolism. A coronary steal of blood flow away from ischaemic areas may occur because
nitroprusside dilates resistance vessels in nonischaemic myocardium, causing diversion of blood flow from ischaemic
areas where vessels are maximally dilated. Attenuation of hypoxic pulmonary vasoconstriction may accompany
infusions of nitroprusside with an associated decrease in the arterial PO2. Positive end-expiratory pressure (PEEP)
may reverse vasodilator induced reductions in the arterial PO2. Infusions of nitroprusside delivering more than 16mg
may result in dose-related decreases in platelet aggregation and an increase in bleeding time. The release of rennin
may accompany the reduction in blood pressure induced by nitroprusside and may contribute to the overshoot in
blood pressure when the drug is discontinued. This overshoot may be prevented by administering an infusion of an
angiotensin II inhibitor (e.g. saralasin).

Review Question
Regarding oxytocin, which of the following statements is/are true?
True / False
Causes relaxation of myoepithelial cells in mammary glands Correct

Has 10% of the antidiuretic activity of ADH (antidiuretic hormone) Correct

Is synthesised in the posterior pituitary gland Correct

Lowers the threshold for depolarisation of the uterine smoothmuscle Correct

The sensitivity of the uterus to oxytocin decreases as pregnancy progresses Correct

Oxytocin is synthesised in the paraventricular nucleus of the hypothalamus, and is then transported in
secretory granules along axons to the posterior pituitary gland, from which it is released.

The primary role of oxytocin is to eject milk from the lactating mammary gland in response to suckling. It
achieves this by causing contraction of the myoepithelial cells that surround the alveoli of the mammary
gland.

The principal clinical uses of synthetic oxytocin are to induce labour at term and to induce sustained
uterine contraction which is required for post-partum or post-termination haemostasis.

By lowering the threshold for depolarisation of uterine smooth muscle, oxytocin exerts a contracting effect
on the gravid uterus. The sensitivity of the uterus to oxytocin increases as the pregnancy progresses.

Oxytocin has only 0.5 - 1% of the antidiuretic activity of ADH (antidiuretic hormone) when administered in
high doses, introducing the possibility of water intoxication.

Review Question
The following cause contraction of bronchial smooth muscle:
True / False

Epinephrine Correct

Vasoactive intestinal polypeptide (VIP) Correct

Acetylcholine Correct

Sulphur dioxide Correct

Leukotrienes Correct

Stimulation of sensory receptors in the airways by irritants produces reflex bronchoconstriction, which is
mediated via cholinergic pathways. Thus sulphur dioxide, acetylcholine and Leukotrienes cause
bronchoconstriction. Vasoactive intestinal polypeptide (VIP) relaxes bronchial smooth muscle and
epinephrine is an agonist at beta-2 adreneric receptors.

Review Question
Disopyramide:
True / False

is a class IB antidysrhythmic drug Correct

has an elimination half-life of 2 – 4 hours Correct

commonly causes a dry cough Correct

has membrane stabilising properties Correct

can induce a bi-directional block Correct


Disopyramide is a group Ia antidysrhythmic drug (together with quinidine and procainamide), that has
membrane stabilizing properties. It acts by blocking (open) voltage-dependent sodium channels. The
dominant electrophysiological properties of group Ia drugs are related to their ability to block the rapid
influx of sodium ions during phase 0 depolarisation of the cardiac action potential. This effect causes a
decreased level of membrane responsiveness and slowed conduction of cardiac impulses. These drugs
also decrease the rate of spontaneous phase 4 deploarisation, resulting in reduced automaticity. Group Ia
drugs also induce a bi-directional block and thus interrupt re-entry. Approximately 90% of an oral dose is
absorbed and the elimination half-life is 8 – 12 hours. About 50% is excreted unchanged by the kidney, so
a prolonged elimination half-life is seen in the presence of renal dysfunction. Disopyramide has a negative
inotropic action and may cause hypotension and aggravate heart failure. It also has marked
anticholinergics side effects that include a dry mouth, blurred vision and occasionally urinary retention (not
dry cough).

Review Question
Saturated vapour pressure (SVP):
True / False

when the saturated vapour pressure equals atmospheric pressure the vapour freezes Correct

reducing the temperature of the liquid increases the kinetic energy of the molecules Correct

indicates the degree of volatility Correct

a liquid with a high saturated vapour pressure is easier to vaporise Correct

for volatile agents saturated vapour pressures are quoted at 15 degrees Celsius Correct

The saturated vapour pressure (SVP) is the pressure exerted by the vapour phase of a substance when in
equilibrium with the liquid phase (when the number of molecules leaving the liquid phase equals the
number entering it). It indicates the degree of volatility and agents with higher SVPs are more volatile and
easier to vaporise than agents with lower SVPs. When the SVP equals the atmospheric pressure the
liquid boils (not freezes). The SVP increases with temperature because the temperature rise increases the
kinetic energy of the molecules allowing more molecules to enter the gaseous phase. The SVPs of volatile
agents are usually quoted at standard temperature 20 degrees Celsius (not 15 C).

Review Question
The most likely cause of an alveolar-arterial (A-a) oxygen difference of 30 kPa
in a normal healthy adult breathing 100% oxygen is:
True / False

hypoventilation Correct

atelectasis Correct

a right to left shunt Correct

a diffusion defect in oxygen transport Correct

high altitude Correct

The alveolar-arterial (A-a) oxygen difference is the „ideal‟ alveolar PO2 minus arterial PO2. The „ideal‟
alveolar PO2 is the PO2 which the lung would have if there were no ventilation-perfusion (V/Q) inequality
and it was exchanging gas at the same respiratory exchange ratio as real lung, and is derived from the
alveolar air equation. Arterial PO2 is measured directly. The A-a oxygen difference (or gradient) is a useful
measure of shunt and V/Q mismatch, and in normal adults breathing air the A-a difference is less than 2
kPa (15mmHg). When breathing 100% oxygen the A-a difference increases, because the shunt
component is not corrected and it may be up to 15kPa (115 mmHg). An increased A-a difference may be
caused both by abnormally low and abnormally high V/Q ratios within the lung, though chiefly the former.
The most likely cause of an A-a difference in a healthy adult breathing 100% oxygen is atelectasis, which
would give a low V/Q ratio. Hypoventilation may result in a rise in alveolar (and hence arterial) CO2 which
according to the alveolar air equation may lower the alveolar PO2. High altitude will also lower the alveolar
PO2. A right to left shunt and an oxygen transport diffusion defect are unlikely to be found in healthy
individuals.

Review Question
The following drugs inhibit enzymes:
True / False

Acetazolamide Correct

Allopurinol Correct

Piroxicam Correct

Trandolapril Correct

Cimetidine Correct

Acetazolamide is a carbonic anhydrase inhibitor and a weak diuretic. It is used for prophylaxis against
mountain sickness and in the management of glaucoma. Allopurinol inhibits the enzyme xanthine oxidase
which is responsible for urate synthesis and is used in the management of gout. Piroxicam is a non-
steroidal anti-inflammatory drug (NSAID) that inhibits cyclo-oxygenase. Trandolapril is an angiotensin-
converting enzyme inhibitor (ACE inhibitor). Cimetidine is a histamine type-2 receptor antagonist but as a
side effect it inhibits the mixed function oxidases (cytochrome P-450s) and so retards oxidative hepatic
drug metabolism (phase 1 reactions).

Review Question
Flumazenil:
True / False

is a weak acid Correct

is 90% protein bound Correct

is predominantly bound to albumin Correct

has an initial half life of 20 minutes Correct

has an approximate volume of distribution at steady state of 1 litre / kg Correct

Flumazenil is a benzodiazepine antagonist and is a weak lipophilic base (not acid). It is about 50% bound
to plasma proteins (not 90%) and albumin accounts for about 65% of this protein binding. Flumazenil is
extensively distributed in the extravascular space and the volume of distribution at steady state is 0.9 - 1.1
l/kg. It has a Half life is 7-15 min (initial), 20-30 min (brain), 40-80 min (terminal). This is shorter than that
of diazepam and midazolam such that there is a risk of patients becoming resedated.

Review Question
The following represent the percentage of vapour metabolised:
True / False

Isoflurane 3% Correct

Halothane 25% Correct

Sevoflurane 2% Correct
Enflurane 7% Correct

Desflurane 0.2% Correct

The approximate percentage metabolism of the volatile agents is as follows: Isoflurane 0.2%; Halothane
20%; Enflurane 2%; Sevoflurane 5%; Desflurane 0.02%. Oxidative metabolism within the liver via the
cytochrome P-450 system accounts for the greater part, but metabolism also occurs to a lesser extent
within the kidneys, lungs and gastrointestinal tract.

Review Question
The realative humidity of air in the alveoli during IPPV depends upon:
True / False

Ambient temperature Correct

Atmospheric pressure Correct

Airway pressure Correct

Body temperature Correct

The alveolar surface area Correct

Absolute humidity is the mass of water vapour present in a given volume of air, which is expressed as
either g /m3 or mg / litre. The maximum amount of water vapour that can be present in a given volume of
air is determined by the temperature (both ambient and body). Thus when the temperature increases the
amount of water which can be present as vapour also increases. Hence fully saturated air at 20 oC
contains about 17 g /m3 whereas at 37 oC it contains 44 g /m3 when saturated.

Review Question
The principle by which a strain gauge works is the:
True / False

Absorption of gas by a crystal Correct

Varying scatter of light molecules of different sizes Correct

Varying resistance of a wire when stretched Correct

Uncoiling of a tube by pressurised gas Correct

Reduction of pressure at a constriction in a tube Correct

When a wire is stretched it becomes longer and thinner and consequently its resistance increases, and
this is the principle by which a strain gauge works. A strain gauge acts as a resistor, which are used in
pressure transducers. In a pressure transducer the diaphragm moves as the pressure changes and this
alters the tension in the resistance wire, thus changing its resistance. The changes in current flow through
the resistor are then amplified and displayed as a measure of pressure change. However, to measure or
monitor these changes in resistance, a Wheatstone bridge is frequently used.

Review Question
Diazepam:
Larger doses are required in renal disease Correct

Has a similar volume of distribution in adults and paediatric patients Correct

Is water soluble Correct

Is highly protein bound Correct

Has a longer half-life than nordiazepam (desmethyldiazepam) Correct

Diazepam is a benzodiazepine that is highly protein bound and insoluble in water. It has a half-life of
between 20-70 hours, but is associated with the formation of active metabolites, e.g. nordiazepam
(desmethyldiazepam) which has an even longer half-life (120 hours). The volume of distribution of
diazepam in paediatric patients is similar to adult values but with a greatly prolonged half life. In renal
disease diazepam may have a more marked effect owing to reduced protein binding and a dose reduction
is appropriate. Due to impaired phase I metabolism in liver disease, there is increased sensitivity to
diazepam and midazolam.

Review Question
Mannitol increases urine output by:
True / False

increasing cortical blood flow Correct

increasing tubular flow Correct

reducing solute absorption Correct

blocking the binding of aldosterone Correct

inhibiting carbonic anhydrase Correct

Mannitol is an osmotic diuretic which is filtered at the glomerulus but not reabsorbed and creates an
osmotic gradient into the tubules. Thus it is excreted with an osmotic equivalent of water (and some
sodium) and has been used in forced diuresis to treat drug overdoses, in cerebral oedema and to maintain
a diuresis during surgery. It is a renal vasodilator and increases the cortical blood flow. It increases tubular
flow that clears cellular debris in the tubules. Mannitol also has oxygen free radical scavenging properties,
which may reduce the risk of an ischaemia-reperfusion injury. Following intravenous administration it may
acutely expand the blood volume and so is rarely used in heart failure.

Review Question
Captopril:
True / False

may cause a persistent productive cough Correct

is safe in patients with angioedema Correct

reduces renovascular resistance Correct

increases renal blood flow Correct

reduces aldosterone release Correct

Angiotensin-converting enzyme inhibitors (ACE inhibitors) inhibit the conversion of angiotensin I to


angiotensin II (and thus reduced aldosterone release). As a consequence both arterial and venous
resistance is lowered and cardiac output is increased. The renovascular resistance falls and renal blood
flow is increased, which results in an increase in sodium and water excretion, contracting the circulating
blood volume and reducing venous return. ACE inhibitors can cause profound hypotension and renal
impairment. A persistent dry cough (not productive) is described as are upper respiratory-tract symptoms
such as sinusitis, rhinitis and sore throat. ACE inhibitors may cause angioedema, which can be delayed
and are thus contraindicated in patients known to have angioedema. They are also contraindicated in
patients with known or suspected renovascular disease, e.g. renal artery stenosis.

Review Question
At a dose of 2.5 – 10 micrograms/kg/minute of Dobutamine, the following are
true:
True / False

There is more chronotropy than inotropy Correct

Renal perfusion is increased Correct

Pulmonary vascular resistance decreases Correct

The systemic vascular resistance will be raised Correct

Direct alpha-1 adrenergic receptor stimulation is mild Correct

Dobutamine is a sympathomimetics amine (ino-dilator) that acts as an agonist principally on beta


adrenergic receptors, impacting on beta-1 receptors in cardiac muscle in a relatively selective fashion. It
also has mild indirect alpha-1 adrenergic receptor stimulating effect, (not direct) which is secondary to
preventing noradrenaline uptake at adrenergic nerve terminals, but this is offset by slightly more potent
beta-2 adrenergic receptor stimulation. The effects of dobutamine include: increased inotropy, increased
cardiac output (which may be responsible for the increased renal perfusion and urine output), and minimal
or modest decreases in systemic and pulmonary vascular resistance. At clinical doses there is minimal
effect on heart rate (chronotropy). It has no action at peripheral dopamine receptors. It is not reported to
induce vasoconstriction, but the vasodilatation due to stimulation of vascular beta-2 receptors may limit its
clinical effectiveness.

Review Question
Full cylinders of gas/vapour at 15°C should have a pressure of:
True / False

4400 kPa with entonox Correct

13700 kPa with oxygen Correct

436 kPa with cyclopropane Correct

723 psi with carbon dioxide Correct

5000 kPa with medical compressed air Correct

Cylinders of compressed vapours and gases have the following pressures when full at 15°C: Oxygen
13700 kPa; Entonox 13700 kPa; Nitrous Oxide 4400 kPa; Medical Air 13700 kPa; Carbon Dioxide 5000
kPa (723 psi); Cyclopropane 436 kPa.

Review Question
Carbamazepine:
True / False
has no active metabolites Correct

has neurotoxic side effects Correct

blocks T-type calcium channels Correct

is structurally related to sodium valproate Correct

has an elimination half-life of 6 hours Correct

Carbamazepine is an anticonvulsant that is structurally related to the antidepressant imipramine, and is


used in the management of epilepsy and chronic pain conditions, e.g. trigeminal neuralgia. It acts by
binding to inactivated (closed) sodium channels, which blocks them from returning to their resting state
(also closed), that they must return to before they can open again. Thus the sodium current is
progressively reduced until it is insufficient to initiate an action potential. Phenytoin and sodium valproate
have similar actions on neuronal sodium channels. Ethosuximide and also sodium valproate block T-type
calcium channels in thalamic neurones. Oral absorption is rapid and plasma protein binding is
approximately 80%. There is a linear increase in plasma concentration with dosage and the elimination
half-life is 13 – 17 hours. It is metabolised in the liver to carbamazepine-10,11-epoxide, which is an active
metabolite. The metabolite contributes to its anticonvulsant action and neurotoxic side effects, which
include sedation, vertigo, ataxia, diplopia, nausea and vomiting. Carbamazepine may enhance the
metabolism of phenytoin, whereas phenobarbital may enhance the metabolism of carbamazepine.

Review Question
Alveolar surfactant:
True / False

helps to prevent alveolar collapse Correct

is decreased in hyaline membrane disease Correct

is increased in hypothyroidism Correct

contains approximately 60% dipalmitoylphosphatidylglycine Correct

is decreased in the lungs of heavy smokers Correct

Surfactant is a surface tension lowering agent found in the fluid lining the alveoli, which prevents collapse
and pulmonary oedema. It is produced by type II alveolar epithelial cells and it is a mixture of lipids and
proteins, in the following proportions: dipalmitoylphosphatidylcholine 62%; phosphatidylglycine (5%); other
phospholipids 10%; neutral lipids 13%; proteins 8%; and carbohydrate 2%. Surfactant deficiency is the
cause of hyaline membrane disease and there is a decrease in surfactant in the lungs of cigarette
smokers. Thyroid hormones increase the number of inclusions in the type II cells and hyaline membrane
disease, due to a deficiency in surfactant, is more common in infants with hypothyroidism.

Review Question
Prior to a skilled voluntary movement, increased neuronal activity is initially
seen in the:
True / False

cerebellum Correct

spinal motor neurons Correct

basal ganglia Correct

cortical association areas Correct


precentral motor cortex Correct
Although much remains unknown about the control of voluntary movement, it is generally accepted that
the commands and hence the initial increase in neuronal activity originates in the cortical association
areas. The planning of movements occurs in the cortex as well as the basal ganglia and lateral cerebellar
hemispheres. Information is funnelled by the basal ganglia and cerebellum to the premotor and motor
cortex via the thalamus. Motor commands from the motor cortex are then relayed in large via the
corticospinal and corticobulbar tracts to the motor neurones in the brain stem.

Review Question
Propofol:
True / False

increases the latency and decreases the amplitude of somatosensory evoked potentials (SSEPs) Correct

intralipid may alter blood coagulation Correct

intra-arterial injection is painless Correct

readily crosses the placenta Correct

the presence of phenols may give the urine a green colour Correct

Propofol (2,6-diisopropylphenol) is an intravenous anaesthetic agent used for both induction and
maintenance of anaesthesia. It produces unconsciousness in one arm-brain circulation time and increases
the latency and decreases the amplitude of somatosensory evoked potentials (SSEPs). Accidental intra-
arterial injection is associated with severe pain, though evidence of subsequent vascular complications is
limited. Prolonged intravenous infusions may result in the excretion of green urine, reflecting the presence
of phenols in the urine, without any alteration in renal function. Propofol does not alter coagulation or
platelet function, but intralipid has been associated with alterations in blood coagulation. Propofol readily
crosses the placenta but is rapidly cleared from the neonatal circulation.

Review Question
Are the following suitable antibiotics for treatingEscherichia coli?
True / False

Amoxicillin Correct

Ampicillin Correct

Cefuroxime Correct

Ciprofloxacin Correct

Co-amoxiclav Correct
Escherichia coli is a Gram-negative bacillus which produces penicillinases, but it remains sensitive to
numerous antibiotics.
Bacterial resistance to antibiotics is a major problem and where possible sensitivities should be checked
before blind treatment of infections is commenced.
Ampicillin and amoxicillin are broad spectrum antibiotics active against non-beta lactamase producing
Gram positive and Gram negative organisms. They are inactivated by penicillinases and since up to 50%
of Escherichia coli strains are now resistant, they would be an unsuitable choice.
Co-amoxiclav consists of amoxicillin and the beta-lactamase inhibitor clavulanic acid. Thus it is active
against beta-lactamase producing bacteria that are resistant to amoxicillin, including strains of Escherichia
coli.
Cefuroxime is a cephalosporin that is resistant to inactivation by bacterial beta-lactamases and so may be
used againstEscherichia coli.
Ciprofloxacin has a 6-fluoro substituent which confers enhanced antibacterial potency against both Gram
positive and Gram negative organisms, including Escherichia coli.

Review Question
J waves may be caused by or be a feature of treatment with:
True / False

Perazine Correct

Chlorpromazine Correct

Ethanol Correct

Pipothiazine Correct

Levomapromazine Correct

Hypothermia is defined as a core temperature of less than 35oC and as the core temperature continues to
fall the cardiac rhythm becomes increasingly unstable, sinus bradicardia gives way to atrial fibrillation
followed by ventricular fibrillation and finally asystole. In hypothermic patients J waves are frequently seen
on the ECG. All of the above drugs are associated with a fall in core temperature or disorders of
temperature regulation and therefore these patients may develop J waves

Review Question
Catecholamine secretion is stimulated by:
True / False

Pain Correct

Hypothermia Correct

Hypoglycaemia Correct

Hypoxaemia Correct

Hypercapnia Correct

Cells in the adrenal medulla synthesize and secrete the catecholamines norepinephrine and epinephrine,
which are stored in electron-dense granules (that also contain ATP and several neuropeptides). Secretion
of these hormones is stimulated by acetylcholine release from preganglionic sympathetic fibers
innervating the medulla. Many types of "stresses" stimulate such secretion, including exercise,
hypoglycemia, pain, hypoxaemia, hypercapnia and trauma. The physiologic consequences of medullary
catecholamine release are justifiably framed as responses which aid in dealing with stress. During mild
hypothermia the arterial concentrations of norepinephrine increase, which induces vasoconstriction

Review Question
The following drugs can be safely administered during breast feeding:
True / False

Terbutaline Correct

Fluoxetine Correct

Valproate Correct

Captopril Correct

Chlorpheniramine Correct
The UK Drugs in Lactation Advisory Service http://www.ukmicentral.nhs.uk/drugpreg lists those drugs
which can be safely used by breast feeding mothers. A robust assessment of the balance of benefit to risk
requires data both on the drug's passage into breast milk and its effects in neonates or infants. New drugs
should be avoided if a widely used and therapeutically equivalent alternative is available, as there is rarely
enough information available on new drugs to allow such an assessment to be made. Neonates
(especially if born prematurely) are at greater risk from exposure to drugs via breast milk, because of their
immature excretory functions and the consequent risk of drug accumulation. It is best to avoid long-acting
preparations, especially those drugs likely to cause serious side effects (e.g. antipsychotic agents), as it is
difficult to time feeds to avoid significant amounts of drug in breast milk. Low levels of terbutaline are
found in breast milk after oral maternal dosing and it is safe. Valproate is safe despite a single case of
petechial rash, but the infant should be monitored for signs of rash. Captopril is safe with low levels of the
drug reported in breast milk. Fluoxetine is not safe due to its long half-life and the risk of infant drug
accumulation. Adverse drug reactions have also been reported for fluoxetine (irritability, reduced weight
gain). Chlorpheniramine is also not safe, due to the risk of drowsiness and poor feeding in the
neonate/infant.

Review Question
The following anti-arrhythmic drugs can be used for treating both broad and
narrow complex tachyarrhythmias:
True / False

Amiodarone Correct

Lidocaine Correct

Adenosine Correct

Verapamil Correct

Esmolol Correct

Tachyarrhythmias have by definition a heart rate greater than 100 beats per minute and can be classified
as broad or narrow depending on the duration of the QRS complex. The normal QRS complex is 0.12
seconds. In general terms 75% of broad complex tachycardias originate in the ventricles (though it may
result from supraventricular rhythms with aberrant conduction), but in a peri-arrest context we assume
they originate from the ventricles. Narrow complex tachycardias are supraventricular in origin. Amiodarone
is a class III anti-arrhythmic drug, which is the only drug that can be used to treat both types of
tachyarrhythmia (including atrial fibrillation). It prolongs phase 3 of the cardiac action potential and has
numerous other effects however, including actions that are similar to those of antiarrhythmic classes Ia, II,
and IV. Amiodarone shows beta blocker-like and calcium channel blocker-like actions on the SA and AV
nodes, increases the refractory period via sodium and potassium channel effects, and slows intra-cardiac
conduction of the cardiac action potential, via sodium-channel effects. Narrow complex tachycardias,
should initially be treated by administering high flow oxygen and using vagal manoeuvres. Adenosine can
be given by rapid intravenous injection and may terminate the dysrhythmia by causing a transient block of
conduction at the AV node. Narrow complex tachycardias which are not associated with adverse signs
can be treated using any of the following: Esmolol, Verapamil, Amiodarone and Digoxin. The management
of atrial fibrillation is different from that previously described. Broad complex tachycardias which do not
have a central pulse (pulseless VT) should be treated following the VF protocol. In the absence of adverse
signs and following the administration of high flow oxygen, broad complex tachycardias may initially be
treated with either Amiodarone or Lidocaine. Electrolytes should be within the normal ranges and if
necessary DC cardioversion may be required

Review Question
Which of the following are potential causes of pulseless electrical activity
(PEA):
True / False
type I respiratory failure Correct

a core temperature of 35.5o Correct

hypocalcaemia Correct

a small apical pneumothorax Correct

hypokalaemia Correct

The causes of pulseless electrical activity (PEA), which was previously known as electromechanical
dissociation (EMD), can be summarised into the 4 Hs and 4 Ts, i.e. hypoxia, hypovolaemia, hypothermia,
hyper / hypokalaemia, tension pneumothorax, cardiac tamponade, thromboembolism and toxic /
therapeutic disorders. Type I respiratory failure is characterised by hypoxaemia and a normal or low
PaCO2. Hypothermia is defined as a core temperature less than 35o Celsius. A small pneumothorax,
which is not pressurised, is unlikely to cause hypoxaemia or compromise gas exchange. Low serum
potassium and calcium are recognised causes of PEA

Review Question
Do features of amniotic fluid embolism include the following?
True / False

Abnormal bleeding Correct

Bronchospasm Correct

Cardiorespiratory collapse Correct

Convulsions Correct

Pulmonary hypertension Correct


All the above may be features of amniotic fluid embolism if sufficiently large.
It is an extremely rare emergency condition when amniotic fluid, fetal cells, hair or other debris enters the
mother's blood stream via the placenta and triggers an allergic reaction. This allergic reaction then results
in cardiorespiratory collapse.
Phase one comprises acute dyspnoea and hypertension which may quickly progress to cardiorespiratory
arrest.
Although many women do not survive beyond Phase 1, Phase 2 is also known as the haemorrhagic
phase and may be accompanied by severe shivering, coughing, vomiting and the sensation of a bad taste
in the mouth. It is accompanied by fetal distress and bradycardia together with maternal haemorrhage.

Review Question
Gastric emptying is delayed by:
True / False

pregnancy Correct

dopamine Correct

alcohol Correct

anticholinergics Correct

opioids Correct

The rate of gastric emptying is important because of the risk of regurgitation and aspiration. In addition to
most of the above, (dopamine, alcohol, anticholinergics and opioids) gastric emptying is classically
delayed in labour and usually in pregnancy from the third trimester onwards. To say that gastric emptying
is delayed throughout pregnancy as a whole is false. Gastric emptying is also slowed or delayed by pain,
fear and anxiety. Prokinetic drugs like metoclopramide, erythromycin and cisapride promote gastric
emptying

Review Question
Paramagnetic oxygen analyser:
True / False

Do not require calibration Correct

Utilises null deflection Correct

Can measure gases dissolved in blood Correct

can be used to measure the concentration of Diamagnetic gases Correct

The readings are unaffected by water vapour Correct

The electrons in the outer shell of an oxygen molecule are unpaired, thus it has paramagnetic properties
and is attracted into a magnetic field. Since most other gases are weakly diamagnetic they are repelled by
a magnetic field (nitric oxide is also paramagnetic). Null deflection is a crucial principle in paramagnetic
analysers (reflected beam of light on two photocells) which gives very accurate results (typically 0.1%).
Water vapour affects the readings and for accurate analysis the sample gas must be dried before passing
into the analysis cell, e.g. by passage through silica gel. Therefore, they are unsuitable to measure gases
dissolved in blood. As with most measurement instruments paramagnetic analysers must be calibrated
before use.

Review Question
Bispectral index (BIS):
True / False

Burst suppression patterns are evident at levels below 65 Correct

Is reduced by intraoperative opioids Correct

Inhalational agents lower BIS more than induction agents Correct

Values are expressed in Hz Correct

Predicts the incidence of post-anaesthetic recall Correct

Bispectral index (BIS) monitors are neurophysiologic monitoring devices and the BIS number / value is
derived from a complex mathematical process to determine the harmonic and phase relations among the
various electroencephalogram frequencies. Titrating anaesthetic agents to a specific BIS value during
general anaesthesia may allow the use of less agent, resulting in faster recovery. One may conclude,
incorrectly, that BIS monitoring also reduces the incidence of intraoperative awareness (post-anaesthetic
recall). The incidence of awareness is relatively low and awareness has been reported in the presence of
BIS monitoring, showing that the efficiency of awareness prevention by the BIS is not 100%. The BIS
value is a dimensionless nonlinear parameter which ranges from 0 to 100. A BIS value of 0 equals no
EEG activity and a BIS of 100 defines the awake state. In general, the BIS reflects the awake state at
values exceeding 95, a state of sedation at values of 65 to 85, an arousal state depression suited for
general anaesthesia at values of 40 to 65 and burst suppression patterns becoming evident below values
of 40. The effect of anaesthetic agents on BIS appears to be agent specific. The intravenous anaesthetic
agents (propofol, thiopental and midazolam) have a strong depressant effect on the BIS value, whereas
inhalational agents propagate an intermediate depressant effect on BIS. Opioids have little or no influence
on the BIS at clinically relevant concentrations. Interestingly nitrous oxide and ketamine which deepen the
anaesthetic, appear to have a paradoxical effect and increase the BIS value. Thus BIS monitoring should
not be used when these two agents are in use. Hypothermia also decreases BIS by 1.12 per 0C decline in
body temperature.
Review Question
Urine:
True / False

Normal urine production is approximately 1 ml / Kg per day Correct

Oliguria is defined as urine production < 0.5 ml / Kg per hour Correct

Large doses of intravenous propofol may turn the urine green Correct

Urine is yellow due to the pigments urochrome and uroerythrin Correct

Urine normally contains bilirubin Correct

Urine is coloured yellow by the pigments urochrome and uroerythrin, but urine darkens on standing due to
the oxidation of urobilinogen to urobilin. Abnormal constituents of urine include glucose, ketones, bilirubin,
erythrocytes, large numbers of leucocytes and casts. The urine of patients on long term sedation using
propofol is frequently coloured green. Normal urine output in temperate climates is 800 – 2500 ml / day,
which is about 1 ml/kg per hour. Despite the concentrating ability of the kidney, a minimum of 500 ml/day
is required to eliminate the urea and other electrolytes. Oliguria is defined as a urine production < 0.5
ml/Kg per hour (approx less than 50mls).

Review Question
Are the following known to release histamine?

True / False

Atracurium Correct

Etomidate Correct

Hyoscine Correct

Loratadine Correct

Morphine Correct
Histamine is released from mast cells following the administration of certain drugs, for example, morphine,
d-tubocurare, atracurium. The release of histamine may cause skin erythema and wheals, bronchospasm,
hypotension, anaphylaxis and possibly cardiac arrest.
Loratadine is a H1 receptor antagonist (antihistamine).
Hyoscine is a sedating anticholinergic drug derived from the henbane plant, which only very rarely triggers
histamine release.

Review Question
The following decreases minimal alveolar concentration:
True / False

Clonidine Correct

Neostigmine Correct

Amphetamine Correct

Methyldopa Correct

Reserpine Correct
The minimal alveolar concentration (MAC) of inhalational anaesthetic agents is reduced by CNS
depressant drugs (e.g. opioids, sedatives, other inhalational agents) and by drugs that cause depletion of
CNS catecholamines (e.g. alpha-methyldopa, reserpine). Clonidine is a presynaptic alpha 2 receptor
agonist which suppresses catecholamine release in the CNS by a negative feedback effect; it lowers MAC
and causes sedation. Amphetamines are related to adrenaline and cause stimulation of the CNS and
sympathetic nervous system thus increases MAC. Neostigmine is a quaternary ammonium compound and
crosses the blood brain barrier (BBB) poorly and has few CNS effects.

Review Question
Regarding opioid receptors:
True / False

They are intracellular structures Correct

They contain between 3500 and 4000 amino acids Correct

In the spinal cord, kappa receptors are predominantly localised in lamina IX Correct

Binding with an opioid agonist increases potassium conductance Correct

Binding with an opioid antagonist inhibits voltage sensitive calcium channels Correct

Opioid receptors are members of the large family of seven transmembrane domain receptors (rather than
just intracellular), coupled to pertussis toxin-sensitive G proteins. Four types have been characterised and
cloned and designated delta (DOR), mu (MOR), kappa (KOR) and orphan receptor-like 1 (ORL-1). They
each contain between 372 – 400 amino acids and have different molecular weights. Binding of opioid
agonists to the receptors causes: inhibition of adenyl cyclase (reducing cAMP levels); an increase in
potassium conductance (the G-protein-activated inwardly rectifying conductance or GIRK); inhibition of
voltage sensitive calcium channels; and a decrease in transmitter release (excitatory and /or inhibitory).
Protein kinase C is also activated. Opioid receptors are located in plasma membranes of neuronal and
non-neuronal cells located at pre, post and extra-synaptic sites. In the CNS they are differentially
distributed in the brain and spinal cord. The expression of kappa receptors (KOR) in the spinal cord is high
in laminae I and II, and moderate throughout laminae III – VIII and X. Delta receptors (DOR) are
predominantly in lamina IX (not Kappa) with lesser densities throughout laminae III and VIII. Mu receptors
(MOR) are predominantly localised in laminae I and II with moderate expression throughout laminae III –
VIII and X.

Review Question
The following substances modulate peripheral nociceptive input in the dorsal
horn:
True / False

Serotonin Correct

Noradrenaline Correct

Glutamate Correct

Adenosine Correct

Substance P Correct

The dorsal horn is the site where primary afferents terminate, and there is a complex interaction
between these afferent fibres, local intrinsic spinal neurones and descending fibres from the brain.
A number of substances (peptides, catecholamines and indoleamines) have been implicated as
neurotransmitters at the dorsal horn, and when released they modulate peripheral nociceptive
input. They include: substance P, serotonin, noradrenaline, acetylcholine, glutamate and
adenosine. Reflex activity also modulates peripheral nociception.

Review Question
The following may reduce the pain on injection associated with intravenous
propofol administration:

True / False

Metoclopramide Correct

Neostigmine Correct

Ketorolac Correct

Ephedrine Correct

Magnesium Sulphate Correct

A major disadvantage of intravenous propofol administration is the pain on injection. Pre-treatment with
lidocaine, opioids, ephedrine, metoclopramide, ketorolac, neostigmine, metoprolol and magnesium
sulphate have all been reported to reduce the incidence and severity of pain. Contact time with the
endothelium has been implicated though there is conflicting data regarding the influence of speed of
injection on pain. Further information on this topic may be found in: 1) Scott RP, et al. Propofol: clinical
stratergies for preventing the pain of injection. Anaesthesia 1988; 43: 492-4. 2) Picard P, et al. Prevention
of pain on injection with propofol: a quantitative systematic review. Anesth Analg 2000; 90: 963-9. 3) Asik
I, et al. Pain on injection of propofol: a comparison of metoprolol with lidocaine. Eur J Anaesthesiol 2003;
20: 487-9.

Review Question
The following are contraindicated in patients receiving Monoamine-oxidase
inhibitors (MAOIs):
True / False

Dopamine Correct

Ephedrine Correct

Dopexamine Correct

Dexamfetamine Correct

Pethidine Correct

Monoamine-oxidase inhibitors (MAOIs) are a class of antidepressant which are prescribed less frequently
than other classes due to their association with dangerous dietary and drug interactions. The inhibition of
the enzyme monoamine-oxidase causes an accumulation of amine neurotransmitters resulting in CNS
and CVS effects. Drugs that should be avoided include: Adrenaline, noradrenaline, ephedrine,
pseudoephedrine, phenylephrine, dopamine, dopexamine, dexamfetamine and pethidine. The metabolism
of sympathomimetics is inhibited which may potentiate their pressor action, and the pressor effect of
tyramine (found in mature cheese, meat and yeast extracts) may also be potentiated. Alcoholic and low
alcohol drinks should also be avoided. Patients taking MAOIs should not be given pethidine as this is
associated with a drug interaction causing CNS and CVS side effects.

The danger of interaction persists for up to 2 weeks after treatment with MAOIs is discontinued, so drug
withdrawal (if appropriate) should be done slowly.
Review Question
Phenytoin:
True / False

Is secreted primarily in the urine Correct

Is highly protein bound Correct

Causes enzyme inhibition Correct

Has >70% bioavailability following oral administration Correct

May cause gingival hyperplasia Correct

First synthesized in 1908, phenytoin (diphenylhydantoin) is a commonly used anticonvulsant. It is related


to the barbiturates in chemical structure, but has a five-membered ring, with the chemical name of 5,5-
diphenyl-2,4-imidazolidinedione. It has a molecular weight of 252 and a 70 to 100% bioavailability
following oral administration (25% after rectal dosage). A highly protein bound drug (70 to 95%), which is
metabolised by the liver (non-linear) and genetic variation in the gene controlling metabolism occurs
(CYP2C9 and CYP2C19). It has an elimination half life of 22 hours and is excreted primarily through the
bile (<5% in the urine) but its metabolites undergo reabsorption and then secretion in urine. Phenytoin,
phenobarbital (and primidone), carbamazepine, and ethosuximide are all enzyme-inducing
anticonvulsants. The side effects and drug interactions of anticonvulsants are mediated indirectly through
their effects on hepatic microsomal enzyme (P450) systems. Pharmacologic induction of these enzymes
may enhance the metabolism of both endogenous and exogenous substances, including other
anticonvulsants, reducing their plasma concentrations

Review Question
Alkalosis may be caused by:
True / False

Magnesium hydroxide Correct

Acetazolamide Correct

Bendroflumethiazide Correct

Penicillin Correct

Hydrochloric acid Correct

Metabolic alkalosis is a primary increase in serum bicarbonate (HCO3-) concentration, which occurs as a
consequence of a loss of H+ from the body or a gain in HCO3-. There are many causes of metabolic
alkalosis but they can be divided into chloride-responsive alkalosis (urine chloride <20 mEq/L), chloride-
resistant alkalosis (urine chloride >20 mEq/L), and other causes, including alkali-loading alkalosis.
Ingestion of large doses of non-absorbable antacids (eg, magnesium hydroxide) may generate metabolic
alkalosis by a rather complicated mechanism. Upon ingestion of magnesium hydroxide, calcium, or
aluminum with base hydroxide or carbonate, the hydroxide anion buffers hydrogen ions in the stomach.
The cation binds to bicarbonate secreted by the pancreas, leading to loss of bicarbonate with stools. In
this process, both hydrogen ions and bicarbonate are lost, and, usually, no acid-base disturbance occurs.
Sometimes, not all the bicarbonate binds to the ingested cation, which means that some bicarbonate is
reabsorbed in excess of the lost hydrogen ions. This occurs primarily when antacids are administered with
a cation-exchange resin (eg, sodium polystyrene sulfonate); the resin binds the cation, leaving
bicarbonate unbound. The most common causes of metabolic alkalosis are the use of diuretics and the
external loss of gastric secretions. Bendroflumethiazide) and loop diuretics enhance sodium chloride
excretion in the distal convoluted tubule and the thick ascending loop, respectively (but the urine chloride
may not be < 20 mEq/L). These agents thus cause metabolic alkalosis by chloride depletion and by
increased delivery of sodium ions to the collecting duct, which enhances potassium ion and hydrogen ion
secretion. Volume depletion also stimulates aldosterone secretion, which enhances sodium ion
reabsorption in the collecting duct and increases hydrogen ion and potassium secretion in this segment.
Urine chloride is low after discontinuation of diuretic therapy, while it is high during active diuretic use. The
administration of penicillin, carbenicillin, or other semisynthetic penicillins may cause hypokalaemic
metabolic alkalosis by distal delivery of non-reabsorbable anions with an absorbable cation such as Na+.

Review Question
Hyoscine hydrobromide:
True / False

Has an antiemetic action Correct

Causes sedation Correct

Does not cause pupillary dilatation Correct

Rarely causes a tachycardia Correct

Crosses the Blood Brain Barrier Correct

The antimuscarinic drug hyoscine (scopolamine) is a naturally occurring belladonna alkaloid with smooth
muscle relaxant (antispasmodic) and antisecretory properties. Hyoscine hydrobromide does cross the
blood-brain barrier and so may cause sedation or drowsiness (hyoscine butylbromide does not cross the
BBB) and it has a central anti-emetic action (useful in motion sickness). Repeated administration of
hyoscine hydrobromide leads to accumulation, which occasionally can paradoxically result in an agitated
delirium. The central anticholinergic syndrome (excitement, ataxia, hallucinations, behavioural
abnormalities and drowsiness) is a side effect of its use. Pupillary dilatation is a feature, but a tachycardia
(heart rate >100/minute) is rarely seen.

Review Question
Hypothermia may be caused by or be a feature of
True / False

Erythroderma Correct

Arginosuccinic aciduria Correct

Menke disease Correct

Immobility Correct

Kwashiorkor Correct

Erythroderma (also called red man syndrome) is an inflammatory skin disease and one of its features is
abnormal temperature control resulting in fever and chills or hypothermia. Argininosuccinic aciduria is an
autosomal recessive disorder of the urea cycle, caused by mutation in the gene encoding
argininosuccinate lyase. Neuropathological changes can occur in argininosuccinic aciduria, with astrocyte
transformation to Alzheimer type II glia Menke disease (also known as kinky hair disease) is an X-linked
neurodegenerative disease of impaired copper transport. Copper is a trace metal in many essential
enzyme systems, including cytochrome C oxidase, superoxide dismutase, lysyl oxidase, tyrosinase,
ascorbic acid oxidase, ceruloplasmin, and dopamine beta hydroxylase. The deficiency or impaired
function of these enzyme systems is thought to be responsible for the clinical findings in Menke disease.
Temperature instability is an associated feature of this disease. Immobility is a common problem in the
elderly and infirm and this is a contributing factor to their susceptibility to hypothermia. Malnutrition
represents the cellular imbalance between the supply of nutrients and energy and the body's demand for
them to ensure growth, maintenance, and specific functions. The term protein-energy malnutrition (PEM)
applies to a group of related disorders which includes kwashiorkor and represents a maladaptive response
to starvation. Hypothermia is a feature of this disease. Hypothyroidism and anorexia are also associated
with hypothermia.

Review Question
Patients fitted with permanent cardiac pacemakers can safely have the
following investigations:
True / False

CT scan Correct

12 lead ECG Correct

MRI scan Correct

Chest X-ray Correct

V/Q (ventilation / perfusion) Correct

Patients fitted with permanent cardiac pacemakers should never be exposed to the magnetic field of an
MRI scanner. A magnet placed over older pacemakers could change the programming to create a fixed
rate device, but modern pacemakers do not have this failsafe option and the programming can be
scrambled by a small magnetic field. In order to avoid pacemaker malfunction and injury to the patient an
MRI scan is contraindicated, but none of the other investigations are contraindicated. Important
information about the intrinsic cardiac rhythm, the type of pacemaker and how it has been programmed
can be obtained from a 12 lead ECG. A chest x-ray can be used to confirm the location of the box and
location and number of the pacemaker wires within the heart.

Review Question
Remifentanil hydrochloride:
True / False

has a pH of approximately 3 Correct

has a pKa of 7 Correct

has a clearance of 400 ml/min/kg Correct

is approximately 70% bound to plasma proteins Correct

contains 15mg of glycine Correct

Remifentanil hydrochloride is an opioid agonist (molecular weight 412.91) with rapid onset and peak effect
and ultra-short duration of action. Each vial contains Remifentanil base with 15mg glycine and
hydrochloric acid to buffer the solutions to an approximate pH of 3 after reconstitution (pH range is 2.5 –
3.5). It has a pKa of 7.07. Remifentanil is approximately 70% bound to plasma proteins of which two-thirds
is binding to alpha-1-acid-glycoprotein. The initial volume of distribution (Vd) of remifentanil is
approximately 100 ml/kg and represents distribution throughout the blood and rapidly perfused tissues.
Remifentanil subsequently distributes into peripheral tissues with a steady-state volume of distribution of
approximately 350 ml/kg. These two distribution volumes generally correlate with total body weight (except
in severely obese patients when they correlate better with ideal body weight [IBW]). Remifentanil is an
esterase-metabolized opioid. A labile ester linkage renders this compound susceptible to hydrolysis by
non-specific esterases in blood and tissues. The carboxylic acid metabolite is essentially inactive (1/4 600
as potent as remifentanil in dogs) and is excreted by the kidneys with an elimination half-life of
approximately 90 minutes. Remifentanil is not metabolized by plasma cholinesterase
(pseudocholinesterase) and is not appreciably metabolized by the liver or lung. The clearance of
remifentanil is approximately 40 ml/min/kg, and combined with a relatively small volume of distribution
produces a short elimination half-life of approximately 3 to 10 minutes. This value is consistent with the
time taken for blood or effect site concentrations to fall by 50% (context-sensitive half-times) which is
approximately 3 to 6 minutes. Assays of histamine in patients and normal volunteers have shown no
elevation in plasma histamine levels after administration of remifentanil.

Review Question
Haematocrit:
True / False

is higher in venous blood than in arterial blood Correct

tissue oxygen delivery is increased when the haematocrit is below 30% Correct

the optimal value following haemorrhage is approximately 30% Correct

the viscosity of blood is reduced when the haematocrit is high Correct

the haematocrit should be expressed as a percentage Correct

The haematocrit is the total red blood cell volume as a proportion of blood volume and it is expressed as a
fraction not as a percentage. Normal values are 0.4 – 0.54 (male) and 0.37 – 0.47 (female). Venous blood
has a higher haematocrit than arterial blood because of the entry of chloride ions into red cells (chloride
shift) which is followed by water entry by osmosis. A fall in haematocrit decreases the viscosity and thus
increases the flow. Therefore, a haematocrit of about 0.3 after acute blood loss is thought to be optimal
(not 30%); in addition to reducing the viscosity and improving tissue blood flow the hazards of blood
transfusion and deep vein thrombosis are reduced. However, a value below 0.3 is undesirable because of
reduced oxygen carrying capacity.

Review Question
The posteroanterior (PA) chest x ray:
True / False

The x ray tube should be 2 m above the patient Correct

The exposure is made during full inspiration Correct

A voltage of 70 kVp will inadequately demonstrate calcification Correct

Mediastinal and lung detail is obtained by using voltages of 125-150 kVp Correct

Using a grid minimises scattered radiation Correct


In a PA (posteroanterior) chest x ray the patient stands erect with the anterior chest wall against the
vertical film holder. The chin is elevated and the shoulders are rotated forwards thus preventing the
scapulae from overlapping the lung fields.
The x ray tube is situated 2m behind (not above) the patient and the beam is aimed at the fourth thoracic
vertebrae. The exposure is made during full inspiration.
A voltage of 70 kVp will give good bony detail and demonstrate calcification. Improved visualisation of
mediastinal and lung detail requires a higher voltage, e.g. 125-150 kVp, but bones and calcification are not
so clearly seen with these higher voltages.
In obese patients, greater detail may be obtained by using a grid to minimise scattered radiation.

Review Question
The absorption of Calcium from the gut:
True / False

depends on the hormone secreted by the parafollicular cells (or C cells) in the thyroid gland Correct
increases in pregnancy Correct

is an active process Correct

occurs primarily in the ileum Correct

is so efficient that little calcium is present in faeces Correct


Calcium is absorbed from the upper small intestine by an active process, but there is also some
absorption by passive diffusion. Between 30 to 80% of ingested calcium is absorbed (variable efficiency)
and approximately 6 mmol is secreted into the intestinal lumen per day, thus about 19 mmol of calcium is
lost in the faeces each day. The active transport of calcium and phosphate from the intestine is increased
by an active metabolite of Vitamin D, called 1,25-dihydrocholecalciferol (or calcitrol). It is derived from
Vitamin D3 by successive hydroxylation in the liver and finally in the proximal tubules of the kidney. In
addition to increasing calcium absorption from the gut, 1,25-dihydrocholecalciferol facilitates calcium
reabsorption in the kidneys. It also acts on bone, where it mobilizes calcium and phosphate through its
effect on osteoclast function and number. Parathyroid hormone (PTH) is a polypeptide hormone
synthesised and secreted by the chief cells of the parathyroid gland. It increases calcium absorption from
the gut, but this action is solely due to the stimulation of 1,25-dihydrocholecalciferol production. PTH acts
directly on bone to increase bone resorption and mobilize calcium. It increases renal reabsorption of
calcium and increases phosphate secretion in the urine. Calcitonin is a polypeptide hormone secreted by
the parafollicular cells (or C cells) of the thyroid gland, which inhibits osteoclast activity and bone
resorption. Plasma calcitonin concentrations are elevated during pregnancy and lactation. In pregnancy it
is believed that calcitonin blocks the action of 1,25-dihydrocholecalciferol on bone, whilst permitting
increased calcium uptake from the gut to occur without loss of mineral from bone.

Review Question
The following reduce blood glucose:
True / False

Diazoxide Correct

Isoprenaline Correct

Tolbutamide Correct

Gliquidone Correct

Glucagon Correct

Tolbutamide and Gliquidone are sulphonylureas, which act by augmenting insulin release and are
effective in non-insulin dependent (type 2) diabetes mellitus. Glucagon is a polypeptide hormone produced
by the alpha cells of the islets of Langerhans, which mobilises hepatic glycogen stores. It can be injected
by any route and is used to treat hypoglycaemia. Diazoxide is an intravenous antihypertensive agent,
which is associated with hyperglycaemia and sodium / water retention. Isoprenaline is a beta-adrenergic
agonist (inotropic sympathomimetic) which increases cardiac output, heart rate and blood glucose levels;
it is only available by special order.

Review Question
Hormones produced by the hypothalamus include:
True / False

Prolactin Correct

Growth hormone Correct

Corticotrophin releasing hormone Correct

Antidiuretic hormone Correct


Thyroid stimulating hormone Correct

The hypothalamic releasing and inhibiting hormones are secreted from the median eminence of the
hypothalamus, which is a region with few nerve cell bodies, but there are many nerve endings close to
capillary loops, which are the origins of the portal vessels. The hormones produced by the hypothalamus
include: corticotrophin-releasing hormone (CRH); thyrotropin-releasing hormone (TRH); growth hormone-
releasing hormone (GRH); growth hormone-releasing hormone (GIH or somatostatin); lutenizing hormone-
releasing hormone (LHRH or gonadotropin-releasing hormone GnRH); and prolactin-inhibiting hormone
(PIH) and a prolactin-releasing hormone (PRH). Prolactin, growth hormone (GH) and thyroid stimulating
hormone (TSH) are anterior pituitary hormones. Antidiuretic hormone (ADH or vasopressin) is produced in
the supraoptic and paraventricular nuclei of the hypothalamus and released from the posterior pituitary.

Review Question
The following statements regarding the classification of opioids are true:
True / False

Buprenorphine is an agonist/antagonist Correct

Naltrexone is a partial agonist Correct

Pentazocine is an agonist at kappa receptors Correct

Tramadol is a partial agonist at mu receptors Correct

Codeine is a weak agonist at delta receptors Correct

The classification of opioid drugs according to their affinity and efficacy for the different opioid receptors is
as follows: Morphine is a strong agonist at the mu and delta receptors. Codeine is a weak agonist at mu
and delta receptors. Fentanyl and Sufentanil are strong agonist at mu. Tramadol is a weak agonist at mu
receptors. Naloxone and naltrexone are both antagonists at the mu, delta and kappa receptors.
Buprenorphine is a partial agonist at the mu and delta. Pentazocine is an example of agonist/antagonist,
and has agonist activity at kappa receptors and antagonist activity at the mu receptors.

Review Question
Local anaesthetic adjuvant drugs include:
True / False

Ziconotide Correct

Clonidine Correct

Adrenaline Correct

Neostigmine Correct

Ketamine Correct

The rationale for combining adjuvant drugs with local anaesthetics is to lower the dose of each agent,
whilst maintaining analgesic efficacy (or to intensify / prolong their effect) and reducing the incidence and
severity of adverse side effects. The adjuvant drugs interact with neurotransmitters at the dorsal horn
systems and block nociceptive transmission in the spinal cord. The most widely used include: alpha 2
adrenergic drugs (clonidine), anticholinergics (neostigmine), NMDA receptor antagonists (ketamine), N-
specific calcium-channel antagonists (ziconotide), opioids and adrenaline.

Review Question
Adenosine receptors:
True / False

Are intracellular receptors Correct

Two classes have been identified Correct

The A1 and A2 receptors are present centrally and peripherally Correct

Agonists at the A1 receptors are algogenic Correct

Agonists at the A2 receptors are antinociecptive Correct

Adenosine receptors are expressed on the surface of most cells, and four classes are known to exist (A1,
A2A, A2B and A3) although a fifth class may recently have been identified. The A1 and A2 receptors are
present both peripherally and centrally, with agonists at the A1 receptors being antinociceptive and
agonists at the A2 receptors being algogenic (i.e. activation results in pain). The role of adenosine and
other A1 receptor agonists is currently under investigation for use in acute and chronic pain states.

Review Question
Inhalational anaesthetic agents have the following boiling points.
True / False

Trichloroethylene 87°C C Correct

Cyclopropane 33°C Correct

Enflurane 56°C Correct

Chloroform 61°C C Correct

Sevoflurane 58°C Correct

The boiling points of the different volatile agents are as follows:


 Trichloroethylene 87°C
 Enflurane 56°C
 Chloroform 61°C
 Sevoflurane 58°C
 Halothane 50°C
 Isoflurane 48°C
 Desflurane 24°C
 Cyclopropane has a boiling point of -33°C.

Review Question
The action of noradrenaline released at sympathetic nerve endings is
terminated by:
True / False

enzymatic decarboxylation Correct

enzymatic inactivation by catechol-O-methyl transferase Correct

re-uptake of noradrenaline by the nerve terminals Correct

oxidative deamination by monoamine oxidase Correct

its removal by the circulating blood Correct


The action of noradrenaline (norepinephrine) released at sympathetic nerve endings is terminated by its
80% active re-uptake by postganglionic nerve terminals for reuse. The remaining 20% is metabolised by
catechol-O methyltransferase (COMT) and monoamine oxidase (MAO). The initial enzymatic action is
deamination (MAO) and methylation (COMT) rather than decarboxylation.

You might also like